1sq

You might also like

Download as pdf or txt
Download as pdf or txt
You are on page 1of 129

Test-1: GS Subject

( Insta Prelims Test Series 2022 ) Total Marks : 200

1. Consider the following statements regarding Vardhanas


1. The founder of the family of Harsha was Pushyabhuti.
2. Pushyabhutis were the feudatories of the Guptas.
3. Aihole inscription of Pulakesin II mentions the defeat of Pulakesin by Harsha.

Which of the statements given above are correct?


A. 1 and 2 only
B. 2 and 3 only
C. 1 and 3 only
D. 1, 2 and 3

2. Consider the following statements regarding Pushyamitra Sunga


1. The founder of the Sunga dynasty was Pushyamitra Sunga, who was the commander-in-chief
under the Mauryas.
2. Pushyamitra was a staunch follower of Buddhism.

Which of the statements given above is/are correct?


A. 1 only
B. 2 only
C. Both 1 and 2
D. Neither 1 nor 2

3. Consider the following statements regarding Satavahanas


1. The founder of the Satavahana dynasty was Simuka.
2. Gautamiputra Satakarni became famous for his book Gathasaptasati, also called Sattasai.
3. Nasik and Nanaghad inscriptions throw much light on the reign of Gautamiputra Satakarni.

Which of the statements given above are correct?


A. 1 and 2 only
B. 2 and 3 only
C. 1 and 3 only
D. 1, 2 and 3

4. Consider the following statements regarding foreign invaders


1. Menander and his dialogues with the Buddhist monk Nagasena was compiled in the Pali work,
Milindapanho (Questions of Milinda).
2. A Greek ambassador Heliodorus became a Vaishnavite and erected the Garuda Pillar at
Besnagar.

Which of the statements given above is/are correct?


A. 1 only
B. 2 only
C. Both 1 and 2
D. Neither 1 nor 2

www.insightsactivelearn.com 1

https://t.me/upsc_pt_mains
Test-1: GS Subject
( Insta Prelims Test Series 2022 ) Total Marks : 200

5. Consider the following statements regarding Kushanas


1. The founder of the Kushana dynasty was Kujula Kadphises.
2. Kanishka was the founder of the Saka era which starts from 78 A.D.

Which of the statements given above is/are correct?


A. 1 only
B. 2 only
C. Both 1 and 2
D. Neither 1 nor 2

6. Consider the following statements regarding Cheras


1. The Cheras ruled over parts of modern Tamil Nadu only.
2. Their important seaports were Tondi and Musiri.
3. The Pugalur inscription of the first century A.D refers to three generations of Chera rulers.

Which of the statements given above is/are correct?


A. 1 only
B. 2 and 3 only
C. 3 only
D. 1, 2 and 3

7. Consider the following statements regarding Cholas


1. Their capital was first located at Uraiyur and then shifted to Puhar.
2. Karikala was a famous king of the Sangam Cholas.
3. Padirruppattu portrays Karikala’s early life and his military conquests.

Which of the statements given above are correct?


A. 1 and 2 only
B. 2 and 3 only
C. 1 and 3 only
D. 1, 2 and 3

8. Which of the following are sources of Gupta period?


1. Devichandraguptam
2. Puranas
3. Mudhrakshasam
4. Meherauli Iron Pillar Inscription

Select the correct answer using the code given below:


A. 1, 2 and 3 only
B. 2, 3 and 4 only
C. 1, 2 and 4 only

www.insightsactivelearn.com 2
Test-1: GS Subject
( Insta Prelims Test Series 2022 ) Total Marks : 200

D. 1, 2, 3 and 4

9. Consider the following statements regarding Fahien’s Visit


1. Fahien visited India during the reign of Chandragupta II.
2. According to him, Buddhism was in a degrading condition in the northwestern India.
3. He came to India by the sea route and returned by the land route.

Which of the statements given above is/are correct?


A. 1 only
B. 2 and 3 only
C. 3 only
D. 1, 2 and 3

10. Consider the following statements regarding Social Life under Guptas
1. During the Gupta period, the caste system became rigid.
2. The practice of Swyamvara was given up.
3. Women were permitted to studying the religious texts like the Puranas.

Which of the statements given above are correct?


A. 1 and 2 only
B. 2 and 3 only
C. 1 and 3 only
D. 1, 2 and 3

11. Which of the following battle ended with the Treaty of Allahabad?

A. Revolt of 1857
B. Battle of Buxar
C. First Anglo-Mysore War
D. Second Carnatic war

12. Consider the following pairs of Astapradhan under Shivaji


administration and their role:
Ashtapradhans: Role:
1. Sumanta – Military commander
2. Amatya – Accountant General
3. Sar-i-Naubat – Master of ceremonies
4. Waqenavis – Intelligence, posts and household affairs

Which of the statement above is/are correct?


A. 1 and 2 only

www.insightsactivelearn.com 3
Test-1: GS Subject
( Insta Prelims Test Series 2022 ) Total Marks : 200

B. 2 and 4 only
C. 3 and 4 only
D. 1 and 4 only

13. Consider the following statements


1. The Chauth and Sardeshmukhi were taxes conceived during the reign of Balaji Bajirao
2. The Sardeshmuki directly went to the King's treasury

Which of the statements given above is/are correct?


A. 1 only
B. 2 only
C. Both 1 and 2
D. Neither 1 nor 2

14. The famous book Akbar Nama was written by

A. Abu’l Fazl
B. Shah Jahan
C. Jahangir
D. Abdul Hamid Lahori

15. Consider the following statements regarding China and Central & Eastern European (CEE) 17+1
1. The 17+1 initiative is a China-led format founded in 2012 in Beijing.
2. Lithuania recently joined this group.
3. Bosnia and Serbia are the two Non-European Union members of this group

Which of the statements given above is/are correct?


A. 1 only
B. 1 and 3 only
C. 3 only
D. 2 and 3 only

16. Consider the following statements regarding National Commission for Protection of Child Rights
(NCPCR)
1. It is a statutory body
2. It works under the aegis of Ministry of Women and Child development
3. Under the RTE Act, 2009, the NCPCR can inquire into complaints about violation of the law.

Which of the statements given above is/are correct?


A. 1 and 2 only
B. 2 only
C. 3 only
D. 1, 2 and 3

www.insightsactivelearn.com 4
Test-1: GS Subject
( Insta Prelims Test Series 2022 ) Total Marks : 200

17. Bhashan Char Island, recently seen in news, is located in?

A. Gulf of Mannar
B. South China Sea
C. Gulf of Oman
D. Bay of Bengal

18. Consider the following statements regarding Digital Tax in India


1. The equalization levy was imposed to give level playing field between Indian businesses who
pay tax in India and foreign e-commerce companies who do business in India but do not pay
any income tax in India
2. Offshore e-commerce firms that sell through an Indian arm will not have to pay equalization
levy.

Which of the statements given above is/are correct?


A. 1 only
B. 2 only
C. Both 1 and 2
D. Neither 1 nor 2

19. Davinci+, Veritas missions, recently announced by NASA is related to which planet?

A. Venus
B. Mercury
C. Mars
D. Asteroid Belt

20. Consider the following statements regarding Nagorno-Karabakh region


1. It is a region located in West Asia
2. Majority of population residing in this region are Afghans

Which of the statements given above is/are correct?


A. 1 only
B. 2 only
C. Both 1 and 2
D. Neither 1 nor 2

21. Consider the following statements regarding Mahendravarman I


1. He was also known as Mamalla, which means ‘great wrestler’.
2. His victory over Pulakesin II in the Battle of Manimangalam near Kanchi is mentioned in
Kuram copper plates.
3. He was converted to Saivism by the influence of the Saiva saint, Appar.

www.insightsactivelearn.com 5
Test-1: GS Subject
( Insta Prelims Test Series 2022 ) Total Marks : 200

Which of the statements given above is/are correct?


A. 1 only
B. 2 and 3 only
C. 3 only
D. 1, 2 and 3

22. Consider the following statements regarding Vedic literature


1. The term ‘Veda’ signifies ‘superior knowledge’.
2. The Atharva Veda consists of various details of rules to be observed at the time of sacrifice.
3. The Brahmanas are the treatises relating to prayer and sacrificial ceremony.

Which of the statements given above are correct?


A. 1 and 2 only
B. 2 and 3 only
C. 1 and 3 only
D. 1, 2 and 3

23. Consider the following statements regarding Aryan culture


1. The principal traits of Aryan culture are set out by Vedic, Iranian, and Greek literary texts.
2. Generally, the texts represent agriculture and pastoralism as the principal sources of
livelihood.
3. They used spoked wheels, and fought with bows and arrows which were placed in quivers.

Which of the statements given above is/are correct?


A. 1 only
B. 2 and 3 only
C. 3 only
D. 1, 2 and 3

24. Consider the following statements regarding Nalanda University


1. Hiuen Tsang gives a very valuable account of the Nalanda University.
2. The medium of instruction was Prakrit.
3. It was a Hinayana University

Which of the statements given above is/are correct?


A. 1 only
B. 2 and 3 only
C. 3 only
D. 1, 2 and 3

25. Consider the following statements regarding Indo-Aryan conflicts


1. Aryans were divided into five tribes called panchajana.
2. The Bharatas and the Purus were the ruling Aryan clans, and they were supported by priest

www.insightsactivelearn.com 6
Test-1: GS Subject
( Insta Prelims Test Series 2022 ) Total Marks : 200

Vasishtha.
3. Battle of Ten Kings was fought on the river Parushni.

Which of the statements given above are correct?


A. 1 and 2 only
B. 2 and 3 only
C. 1 and 3 only
D. 1, 2 and 3

26. Consider the following statements regarding Later Vedic Period


1. Satapatha Brahmana refers to the expansion of Aryans to the eastern Gangetic plains.
2. Pravahana Jaivali’s court, the king of Videha, was adorned by scholar Yajnavalkya.

Which of the statements given above is/are correct?


A. 1 only
B. 2 only
C. Both 1 and 2
D. Neither 1 nor 2

27. Consider the following statements regarding Bimbisara


1. Bimbisara belonged to the Haryanka dynasty.
2. Bimbisara was a contemporary of both Vardhamana Mahavira and Gautama Buddha.
3. He was instrumental in convening the First Buddhist Council at Rajagriha soon after the death
of the Buddha.

Which of the statements given above are correct?


A. 1 and 2 only
B. 2 and 3 only
C. 1 and 3 only
D. 1, 2 and 3

28. Consider the following statements regarding Nandas


1. Dhana Nanda was the founder of Nanda Kingdom.
2. The Hathigumpha inscription of Kharavela of Kalinga refers to the conquest of Kalinga by the
Nandas.
3. The Puranas speak of the extensive conquests made by Mahapadma Nanda.

Which of the statements given above are correct?


A. 1 and 2 only
B. 2 and 3 only
C. 1 and 3 only
D. 1, 2 and 3

29. Consider the following statements regarding Cyrus

www.insightsactivelearn.com 7
Test-1: GS Subject
( Insta Prelims Test Series 2022 ) Total Marks : 200

1. He was the first conqueror who led an expedition and entered into India.
2. He fought the famous battle of Hydaspes against Porus.

Which of the statements given above is/are correct?


A. 1 only
B. 2 only
C. Both 1 and 2
D. Neither 1 nor 2

30. Consider the following statements regarding Kautilya


1. Kautilya was called ‘Indian Machiavelli’.
2. The manuscript of Arthasastra was first discovered by R. Shama Sastri in 1904.
3. The Arthasastra contains 15 books and 180 chapters.

Which of the statements given above are correct?


A. 1 and 2 only
B. 2 and 3 only
C. 1 and 3 only
D. 1, 2 and 3

31. Consider the following statements regarding UN Security Council.


1. UNSC recommends the admission of new Members to UNGA.
2. The UN Charter doesn’t clearly define the functions and powers of the Secretary General
3. More than 60 United Nations Member States have never been Members of the Security
Council.
4. A State which is a Member of the United Nations but not of the Security Council may
participate, with right to vote, in its discussions when the Council considers that country's
interests are affected.

Which of the statements given above is/are correct?


A. 1 and 3 only
B. 1 and 4 only
C. 2, 3 and 4 only
D. 1, 2 and 3 only

32. Consider the following statements


1. In case of non-supply of entitled food grains or meals, there is a provision for food security
allowance to entitled beneficiaries.
2. Every pregnant and lactating mother is entitled to a free meal at the local anganwadi as well
as maternity benefits of Rs.60,000 in instalments under NFSA, 2013
3. NFSA also guarantees age appropriate meal, free of charge through local anganwadi for
children up to 6 months and one free meal for children in age group 6-14 years in schools

Which of the statements given above is/are correct?

www.insightsactivelearn.com 8
Test-1: GS Subject
( Insta Prelims Test Series 2022 ) Total Marks : 200

A. 1 and 3 only
B. 2 and 3 only
C. 1 and 2 only
D. 1, 2 and 3

33. Consider the following statements regarding Group 7 (G7)


1. Canada, China, Germany are the members of G7.
2. It is an intergovernmental organisation formed in 1990’s
3. The major purpose of the G-7 is to discuss and deliberate on security issues.

Which of the statements given above is/are correct?


A. 1 and 2 only
B. 2 and 3 only
C. 2 only
D. None

34. In which of the following areas/fields rare earth metals has its applications?
1. Ceramics
2. Catalysts
3. Glass and Polishing

Select the correct answer using the code given below


A. 1, 2 and 3
B. 1 only
C. 1 and 3 only
D. 2 only

35. Consider the following statements regarding Atlantic Charter


1. It is a joint declaration issued during World War I
2. It is the joint declaration between Germany and United States

Which of the statements given above is/are correct?


A. 1 only
B. 2 only
C. Both 1 and 2
D. Neither 1 nor 2

36. Consider the following statements regarding Breach of privilege motion


1. The parliamentary privileges extend to the President who is also an integral part of the
Parliament.
2. The Speaker/RS chairperson is the first level of scrutiny of a privilege motion.

Which of the statements given above is/are correct?

www.insightsactivelearn.com 9
Test-1: GS Subject
( Insta Prelims Test Series 2022 ) Total Marks : 200

A. 1 only
B. 2 only
C. Both 1 and 2
D. Neither 1 nor 2

37. Consider the following statements regarding NATO Summit


1. It is an intergovernmental military alliance.
2. It was established by Washington treaty.
3. NATO membership is open to any other European state in a position to further the principles
of this Treaty and to contribute to the security of the North Atlantic area.

Which of the statements given above is/are correct?


A. 1 and 2 only
B. 1, 2 and 3
C. 2 and 3 only
D. 2 only

38. Consider the following statements regarding Pakke tiger reserve


1. The main perennial streams in the tiger reserve are the Nameri, Khari and Upper Dikorai.
2. It falls within the Eastern Himalaya Biodiversity Hotspot.

Which of the statements given above is/are correct?


A. 1 only
B. 2 only
C. Both 1 and 2
D. Neither 1 nor 2

39. Consider the following statements regarding Food Safety and Standards Authority of India (FSSAI)
1. Ministry of Food Processing Industries is the Administrative Ministry for the implementation of
FSSAI.
2. It has been established under the Prevention of Food Adulteration Act, 1954.
3. It is responsible for protecting and promoting public health through the regulation and
supervision of food safety.

Which of the statements given above is/are correct?


A. 1 and 3 only
B. 2 and 3 only
C. 3 only
D. 1, 2 and 3

40. Consider the following statements regarding Gharial


1. Gharials live in clear freshwater river systems, congregating at river bends where the water is
deeper.

www.insightsactivelearn.com 10
Test-1: GS Subject
( Insta Prelims Test Series 2022 ) Total Marks : 200

2. A typical gharial will reach 12 to 15 feet in length and weigh up to 2,000 pounds.
3. It is naturally found in Pakistan and India.

Which of the statements given above is/are correct?


A. 1 and 2 only
B. 1 and 3 only
C. 2 only
D. 1, 2 and 3

41. Consider the following statements regarding Buddhism


1. The Buddha (and other teachers) taught orally through discussion and debate.
2. None of the Buddha’s speeches were written down during his lifetime.

Which of the statements given above is/are correct?


A. 1 only
B. 2 only
C. Both 1 and 2
D. Neither 1 nor 2

42. Consider the following statements regarding Al-Biruni


1. Al-Biruni’s Kitab-ul-Hind was written in Persian.
2. He translated the works of Euclid (a Greek mathematician) into Sanskrit.
3. He translated Patanjali’s work on grammar, into Arabic.

Which of the statements given above are correct?


A. 1 and 2 only
B. 2 and 3 only
C. 1 and 3 only
D. 1, 2 and 3

43. Consider the following statements regarding Ibn Battuta


1. Muhammad bin Tughlaq appointed him as the qazi or judge of Delhi.
2. According to Ibn Battuta, travelling was safe under Delhi Sultanate.

Which of the statements given above is/are correct?


A. 1 only
B. 2 only
C. Both 1 and 2
D. Neither 1 nor 2

44. Consider the following statements regarding Imperial Chola


1. The founder of the Imperial Chola line was Vijayalaya.
2. The Uttiramerur inscriptions gives a detailed account of the village administration.

www.insightsactivelearn.com 11
Test-1: GS Subject
( Insta Prelims Test Series 2022 ) Total Marks : 200

3. Rajendra I completed the construction of the famous Rajarajeswara temple or Brihadeeswara


temple at Tanjore.

Which of the statements given above are correct?


A. 1 and 2 only
B. 2 and 3 only
C. 1 and 3 only
D. 1, 2 and 3

45. Consider the following statements regarding Al-Biruni’s view on caste system
1. Al-Biruni tried to explain the caste system by looking for parallels in other societies.
2. He attempted to suggest that social divisions were unique to India.
3. The conception of social pollution, intrinsic to the caste system, was according to him, in
conformity to the laws of nature.

Which of the statements given above is/are correct?


A. 1 only
B. 2 and 3 only
C. 3 only
D. 1, 2 and 3

46. In medieval India, Uluq and Dawa were

A. Grading of horses
B. Postal system
C. Banking system
D. System of hierarchy in military

47. Arrange the following travellers from earliest to latest to arrive in India.
1. Marco Polo
2. Afanasii Nikitich Nikitin
3. Seydi Ali Reis

Select the correct answer using the code given below:


A. 1 2 3
B. 1 3 2
C. 2 1 3
D. 2 3 1

48. Which of the following are sites of Old Stone Age?


1. Soan valley
2. Potwar Plateau
3. Siwalik hills

www.insightsactivelearn.com 12
Test-1: GS Subject
( Insta Prelims Test Series 2022 ) Total Marks : 200

4. Attirampakkam near Chennai

Select the correct answer using the code given below:


A. 1, 2 and 3 only
B. 2, 3 and 4 only
C. 1, 2 and 4 only
D. 1, 2, 3 and 4

49. Consider the following statements regarding Varahamihira


1. Varahamihira declared that the earth was spherical in shape and that it rotates on its own
axis.
2. Varahamihira composed Pancha Siddhantika, the five astronomical systems.
3. His Brihadjataka is considered to be a standard work on astrology.

Which of the statements given above is/are correct?


A. 1 only
B. 2 and 3 only
C. 3 only
D. 1, 2 and 3

50. Which of the following are characteristic features of the Neolithic culture?
1. A different type of stone tools called microliths were used.
2. Mud brick houses were built.
3. Polishing of stone tools
4. Large urns were used as coffins for the burial of the dead.

Select the correct answer using the code given below:


A. 1, 2 and 3 only
B. 2, 3 and 4 only
C. 1, 2 and 4 only
D. 1, 2, 3 and 4

51. Which of the following countries is/are surrounded by the Gulf of Aden?
1. Yemen
2. Ethiopia
3. Somalia

Select the correct answer using the code given below


A. 2 only
B. 1 and 3 only
C. 3 only
D. None

52. Consider the following statements regarding Hallmarking of Gold

www.insightsactivelearn.com 13
Test-1: GS Subject
( Insta Prelims Test Series 2022 ) Total Marks : 200

1. India is the biggest consumer of gold.


2. The Bureau of Indian Standard (BIS) operates gold and silver hallmarking scheme in India.

Which of the statements given above is/are correct?


A. 1 only
B. 1 and 3 only
C. 2 only
D. 2 and 3 only

53. Consider the following statements regarding Neutrino


1. They are one of the most abound particles in the universe.
2. They can even pass through the earth and come out on the other side.
3. Neutrinos carry an electrical charge and are nearly massless.

Which of the statements given above is/are correct?


A. 2 only
B. 1 only
C. 1, 2 and 3
D. 1 and 2 only

54. Consider the following statements regarding Public Accounts Committee (PAC)
1. This committee was set up first in 1921 under the provisions of the Government of India Act of
1919
2. By convention, the chairman of the committee is selected invariably from the Opposition.
3. The PAC is formed every five years.

Which of the statements given above is/are correct?


A. 1 and 2 only
B. 2 and 3 only
C. 3 only
D. None

55. Which of the following is/are the key components of Deep Ocean Mission
1. Development of Ocean Climate Change Advisory Services
2. To explore and identify potential sources of hydrothermal minerals that are sources of
precious metals formed from the earth’s crust along the Indian Ocean mid-oceanic ridges.
3. Development of a component for searching deep-sea flora and fauna, including microbes, and
studying ways to sustainably utilise them.

Which of the statements given above is/are correct?


A. 1 only
B. 1 and 3 only
C. 1, 2 and 3
D. 2 and 3 only

www.insightsactivelearn.com 14
Test-1: GS Subject
( Insta Prelims Test Series 2022 ) Total Marks : 200

56. Consider the following statements regarding the International Labour Organisation (ILO)
1. It was established as an agency for the League of Nations following World War I.
2. It became the first specialised agency of the United Nations (UN) in the year 1946.
3. Global Wage Report is released by ILO

Which of the statements given above is/are correct?


A. 1, 2 and 3
B. 1 only
C. 2 only
D. 1 and 3 only

57. Bosporus strait connects which of the following Sea or Bays


1. Black Sea
2. Sea of Marmara
3. Azov Sea
4. Aegean Sea

Select the correct answer using the code given below:


A. 1 and 2 only
B. 1 and 4 only
C. 2 and 3 only
D. 2 and 4 only

58. Pygmy hogs, sometimes seen in the news, is a/an

A. Endangered Species
B. Vulnerable Species
C. Critically Endangered Species
D. Least Concern Species

59. Gain of function, sometimes seen in the news, is related to

A. Micro organisms
B. Space Science
C. Biodiversity
D. Climate Change

60. Consider the following statements regarding the Mansabdari system


1. It was introduced during the reign of Aurangzeb.
2. It was designated by dual representation - one by personal rank (called zat) and the other by
cavalry rank (called sawar).

Which of the statements given above is/are correct?

www.insightsactivelearn.com 15
Test-1: GS Subject
( Insta Prelims Test Series 2022 ) Total Marks : 200

A. 1 only
B. 2 only
C. Both 1 and 2
D. Neither 1 nor 2

61. Arrange the following mahajanapadas from west to east?


1. Koshala
2. Gandhara
3. Anga
4. Avanti

Select the correct answer using the code given below:


A. 1 2 4 3
B. 2 1 4 3
C. 2 4 1 3
D. 1 2 3 4

62. Consider the following statements regarding Magadhas


1. Between the sixth and the fourth centuries BCE, Magadha became the most powerful
mahajanapada.
2. Initially, Pataliputra was the capital of Magadha and was shifted to Rajagaha.

Which of the statements given above is/are correct?


A. 1 only
B. 2 only
C. Both 1 and 2
D. Neither 1 nor 2

63. Consider the following statements regarding Mauryan Empire


1. Chandragupta Maurya founded the Mauryan Empire.
2. Bindusara was the first ruler who inscribed his messages to his subjects and officials on stone
surfaces.

Which of the statements given above is/are correct?


A. 1 only
B. 2 only
C. Both 1 and 2
D. Neither 1 nor 2

64. Asokan pillar inscriptions have been found in which of the following places?
1. Sanchi
2. Girnar
3. Kandahar

www.insightsactivelearn.com 16
Test-1: GS Subject
( Insta Prelims Test Series 2022 ) Total Marks : 200

4. Sarnath

Select the correct answer using the code given below:


A. 1 and 4 only
B. 2 and 3 only
C. 2, 3 and 4 only
D. 1, 2, 3 and 4

65. Consider the following statements regarding Prayaga Prashasti


1. Prayaga Prashasti was composed to record the achievements of the Shaka ruler Rudradaman.
2. The Prayaga Prashasti mentions that the Sudarshana lake was built by a local governor during
the rule of the Mauryas.

Which of the statements given above is/are correct?


A. 1 only
B. 2 only
C. Both 1 and 2
D. Neither 1 nor 2

66. Consider the following statements regarding terms used in Ancient India
1. Masattuvan and Satthavahas was used to refer to large landowners
2. Vellalar was used to refer to successful merchants.
3. Adimai was used to refer to slaves.

Which of the statements given above is/are correct?


A. 1 only
B. 2 and 3 only
C. 3 only
D. 1, 2 and 3

67. Consider the following statements regarding Punch-marked coins


1. The first gold coins were issued in the first century CE by the Kushanas.
2. The first coins to bear the names and images of rulers were issued by the Indo-Greeks.
3. Tribal republics were denied rights to issue coins.

Which of the statements given above are correct?


A. 1 and 2 only
B. 2 and 3 only
C. 1 and 3 only
D. 1, 2 and 3

68. Consider the following statements regarding gotras


1. Each gotra was named after a Vedic seer and all those who belonged to the same gotra were

www.insightsactivelearn.com 17
Test-1: GS Subject
( Insta Prelims Test Series 2022 ) Total Marks : 200

regarded as his descendants.


2. The members of the same gotra could not marry.
3. Under Satavahanas, women retained their father’s gotras instead of adopting names derived
from their husband’s gotra.

Which of the statements given above are correct?


A. 1 and 2 only
B. 2 and 3 only
C. 1 and 3 only
D. 1, 2 and 3

69. Mlechchhas, a term was which was often used in ancient India, refers to

A. Foreigners
B. Fishermen
C. Spies
D. Tribes and nomads

70. The play is set in the ancient city of Ujjayini during the reign of the King Pālaka, near the end of the
Pradyota dynasty that made up the first quarter of the fifth century BCE. The central story is that of a
noble but impoverished young Brahmin, Cārudatta, who falls in love with a wealthy courtesan or
nagarvadhu, Vasantasenā. Which of the following play is described in the above paragraph?

A. Mudrarakshasa
B. Mālavikāgnimitram
C. Malatimadhava
D. Mrichchhakatika

71. Consider the following statements regarding the land classification during Akbar’s period:
1. Polaj is a land which was annually cultivated for each crop in succession and is never allowed
to lie fallow
2. Chachar is land left out of cultivation for a time, so that it may recover its strength.
3. Parauti is land uncultivated for five years and more
4. Banjar is land that has lain fallow for three or four years.

Select the correct answer using the code given below


A. 1 only
B. 2 and 4 only
C. 3 only
D. 2, 3 and 4 only

72. Who among the following is/are nine distinguished persons of Akbar’s court who were known as
Navratnas or nine jewels?
1. Raja Man Singh

www.insightsactivelearn.com 18
Test-1: GS Subject
( Insta Prelims Test Series 2022 ) Total Marks : 200

2. Tansen
3. Todar Mal
4. Ustad Mansur

Select the correct answer using the codes given below:


A. 1 and 2 only
B. 1, 2 and 3 only
C. 3 and 4 only
D. 1, 2, 3 and 4

73. Consider the following statements regarding the famous Aihole Inscription:

1. It was written in Prakrit script.


2. It talks about the defeat of Harshavardhana by Pulakeshin II
3. It was written by Ravikirti.

Which of the statements given above is/are correct?


A. 1 and 2 only
B. 2 and 3 only
C. 1 and 3 only
D. 1, 2 and 3

74. Consider the following statements regarding the peasants during the Mughal period:
1. Khud-kashta term was used to refer to the non-resident cultivators who belonged to some
other village, but cultivated lands elsewhere on a contractual basis.
2. Pahi-kashta term was used to refer to the residents of the village in which they held their
lands.

Which of the statements given above is/are correct?


A. 1 only
B. 2 only
C. Both 1 and 2
D. Neither 1 nor 2

75. Consider the following statements regarding the Kitab-ul-Hind.


1. It was written by Abdur Razzaq Samarqandi.
2. It was written in Arabic language.

Which of the statements given above is/are correct?


A. 1 only
B. 2 only
C. Both 1 and 2
D. Neither 1 nor 2

www.insightsactivelearn.com 19
Test-1: GS Subject
( Insta Prelims Test Series 2022 ) Total Marks : 200

76. Consider the following statements regarding Purandaradasa:


1. He was a contemporary of Kanakadasa.
2. He formulated the basic lessons of teaching Carnatic music by structuring graded exercises.
3. He introduced the raga Mayamalavagowla as the first scale to be learnt by beginners in the
field

Which of the statements given above is/are correct?


A. 1 and 2 only
B. 2 and 3 only
C. 1 and 3 only
D. 1, 2 and 3

77. Consider the following statements regarding the Bhakti saint Ravidas:
1. He was contemporary of Shankaracharya
2. He opposed caste and gender inequality
3. His devotional songs were included in the Sikh scriptures, Guru Granth Sahib.

Which of the statements given above is/are correct?


A. 2 only
B. 2 and 3 only
C. 1 and 3 only
D. 1, 2 and 3

78. Sattriya dance in modern-form was introduced by:

A. Mirabai
B. Chaitanya Mahaprabhu
C. Shankaradeva
D. Sant Kabir

79. Which of the following was/were the works of Krishnadevaraya?


1. Manucharitam
2. Amukthamalyadha
3. Jambavati Kalyanam
4. Harikathasaram

Select the correct answer using the code given below


A. 1 and 2 only
B. 2 and 3 only
C. 3 and 4 only
D. 1 and 4 only

80. Consider the following statements regarding administration under Aurangazeb

www.insightsactivelearn.com 20
Test-1: GS Subject
( Insta Prelims Test Series 2022 ) Total Marks : 200

1. He started the practice of Jarokhadarshan.


2. He reimposed jizya and pilgrim tax

Which of the statements given above is/are correct?


A. 1 only
B. 2 only
C. Both 1 and 2
D. Neither 1 nor 2

81. Consider the following statements regarding Carbon dating


1. Carbon 14, a radioactive carbon isotope is present in all living objects.
2. The half-life of C14 is 5568 years.
3. Antiquities older than 70 million years can be dated by this method.

Which of the statements given above are correct?


A. 1 and 2 only
B. 2 and 3 only
C. 1 and 3 only
D. 1, 2 and 3

82. Consider the following statements regarding Ashokan inscriptions


1. Brahmi script was written from right to left.
2. The Brahmi script prevailed virtually all over India except for the north-western part.
3. Ashokan epigraphs were first deciphered in 1837 by James Prinsep.

Which of the statements given above are correct?


A. 1 and 2 only
B. 2 and 3 only
C. 1 and 3 only
D. 1, 2 and 3

83. Consider the following statements regarding Jatakas


1. Each birth story of Buddha is called a Jataka.
2. The Jatakas throw invaluable light on the social and economic conditions of the period
between the fifth and tenth century BC.

Which of the statements given above is/are correct?


A. 1 only
B. 2 only
C. Both 1 and 2
D. Neither 1 nor 2

84. Consider the following statements regarding Sangam literature

www.insightsactivelearn.com 21
Test-1: GS Subject
( Insta Prelims Test Series 2022 ) Total Marks : 200

1. The Sangam literature comprises about 30,000 lines of poetry arranged in eight anthologies
called Purananuru.
2. The poems are collected in groups of hundreds such as Ettuttokai.
3. There are two main groups, Patinenkil Kannakku (The Eighteen Lower Collections) and
Pattuppattu (The Ten Songs).

Which of the statements given above is/are correct?


A. 1 only
B. 2 and 3 only
C. 3 only
D. 1, 2 and 3

85. Which of the following species were known to Harrapans?


1. Buffalo
2. Goat
3. Deer
4. Gharial

Select the correct answer using the code given below:


A. 1 and 4 only
B. 2 and 3 only
C. 2, 3 and 4 only
D. 1, 2, 3 and 4

86. Consider the following statements regarding Agricultural technologies under Harapan civilization
1. Terracotta models of the plough have been found at sites in Cholistan and at Banawali.
2. Evidence of a ploughed field was found at Shortughai in Afghanistan.
3. Traces of canals have been found at the Harappan site of Kalibangan.

Which of the statements given above is/are correct?


A. 1 only
B. 2 and 3 only
C. 3 only
D. 1, 2 and 3

87. Consider the following statements regarding Craft Production under Harapan civilization
1. Mohenjodaro was almost exclusively devoted to craft production.
2. Metals like copper, bronze and gold were used for craft production.

Which of the statements given above is/are correct?


A. 1 only
B. 2 only
C. Both 1 and 2
D. Neither 1 nor 2

www.insightsactivelearn.com 22
Test-1: GS Subject
( Insta Prelims Test Series 2022 ) Total Marks : 200

88. Consider the following statements regarding resources during Harapan civilization
1. Lothal was near the best source of lapis lazuli.
2. Nageshwar and Balakot were areas where shell was available.
3. The inhabitants of Khetri region supplied copper to the Harappans.

Which of the statements given above is/are correct?


A. 1 only
B. 2 and 3 only
C. 3 only
D. 1, 2 and 3

89. Consider the following statements regarding the script of Harappans


1. Most inscriptions are short.
2. The script was written from right to left.
3. Motif was used to convey a meaning to those who could not read.

Which of the statements given above is/are correct?


A. 1 only
B. 2 and 3 only
C. 3 only
D. 1, 2 and 3

90. Consider the following statements regarding Ganas


1. Ganas or Sanghas were oligarchies.
2. Both Mahavira and the Buddha belonged to ganas.

Which of the statements given above is/are correct?


A. 1 only
B. 2 only
C. Both 1 and 2
D. Neither 1 nor 2

91. Consider the following statements regarding the Military administration under the Vijayanagar
Empire
1. The top-grade officers of the army were known as Nayaks or Poligars.
2. Quality breed horses were procured from foreign traders
3. Soldiers were usually paid in cash.

Which of the statements given above is/are correct?


A. 1 and 2 only
B. 2 and 3 only
C. 1 and 3 only
D. 1, 2 and 3

www.insightsactivelearn.com 23
Test-1: GS Subject
( Insta Prelims Test Series 2022 ) Total Marks : 200

92. Raya Gopurams and Kalyanamandapam are the distinct feature of :

A. Chalukya Architecture
B. Pallava Architecture
C. Vijayanagara Architecure
D. Gupta Architecture

93. Consider the following statements regarding Hampi:


1. It is located on the banks of the Kaveri River
2. It is a UNESCO world heritage site.

Which of the statements given above is/are correct?


A. 1 only
B. 2 only
C. Both 1 and 2
D. Neither 1 nor 2

94. During the Medieval Indian history, the term ‘Sarraf’, was used to refer:

A. Moneylender
B. Persian Wheel
C. Village Headman
D. In-charge of ports during Akbar period

95. Consider the following statements regarding Mahmud Gawan:


1. Under his rule, the old provinces (tarafs) were consolidated from four into two.
2. The salaries and obligations of each noble were fixed.
3. In every province, a tract of land (khalisa) was set apart for the expenses of the Sultan.

Which of the statements given above is/are correct?


A. 1 and 2 only
B. 3 only
C. 2 and 3 only
D. 1, 2 and 3

96. Karez’ system, a water harnessing technology that originated in Iran/Persia, was built in India by:

A. Cholas
B. Bahmanis
C. Mughals
D. Hoysalas

www.insightsactivelearn.com 24
Test-1: GS Subject
( Insta Prelims Test Series 2022 ) Total Marks : 200

97. Consider the following pairs


Travelers Contemporary Kings
1. Domingo Paes Harshavardhan
2. Barbosa Mahmud Begarha
3. Ibn Battutah Aurangazeb

Which of the pairs given above is/are correctly matched?


A. 1 and 3 only
B. 2 only
C. 2 and 3 only
D. 1 only

98. Consider the following historical battles:


1. Battle of Plassey
2. Third Battle of Panipat
3. Battle of Buxar

Which of the above given events took place during the reign of Mughal emperor Shah Alam II?
A. 1 and 2 only
B. 3 only
C. 2 and 3 only
D. 1 and 3 only

99. Which of the following factors is/are the reasons for the Downfall of the Mughals?
1. The invasions of Nadir Shah and Ahmad Shah Abdali
2. The weak successors after the death of Aurangzeb
3. The financial difficulties due to continuous wars.

Select the correct answer using the codes given below:


A. 2 only
B. 1 and 3 only
C. 2 and 3 only
D. 1, 2 and 3

100. Who among the following Bhakti saints propounded the Pushtimarg philosophy?

A. Shankaracharya
B. Madvacharya
C. Ramanujacharya
D. Vallabacharya

www.insightsactivelearn.com 25
Total Marks : 200
Test-1: GS Subject
( Insta Prelims Test Series 2022 )

1. Consider the following statements regarding Vardhanas


1. The founder of the family of Harsha was Pushyabhuti.
2. Pushyabhutis were the feudatories of the Guptas.
3. Aihole inscription of Pulakesin II mentions the defeat of Pulakesin by Harsha.

Which of the statements given above are correct?


A. 1 and 2 only
B. 2 and 3 only
C. 1 and 3 only
D. 1, 2 and 3

Correct Answer : A

Answer Justification :

Early Life of Harsha

The founder of the family of Harsha was Pushyabhuti.

Pushyabhutis were the feudatories of the Guptas. They called themselves


Vardhanas.

After the Hun invasions they assumed independence.

The first important king of Pushyabhuti dynasty was Prabhakaravardhana.

His capital was Thaneswar, north of Delhi.

He assumed the title Maharajadhiraja and Paramabhattaraka.

The most important military campaign of Harsha was against the Western Chalukya
ruler Pulakesin II. the Aihole inscription of Pulakesin II mentions the defeat of
Harsha by Pulakesin, who after this achievement assumed the title Paramesvara.
Hiuen Tsang’s accounts also confirm the victory of Pulakesin. Hence, statement 3 is
incorrect.

Harsha and Buddhism

In his early life, Harsha was a devout Saiva but later he became an ardent Hinayana

www.insightsactivelearn.com 1
Total Marks : 200
Test-1: GS Subject
( Insta Prelims Test Series 2022 )

Buddhist. Hiuen Tsang converted him to Mahayana Buddhism.

2. Consider the following statements regarding Pushyamitra Sunga


1. The founder of the Sunga dynasty was Pushyamitra Sunga, who was the commander-in-chief
under the Mauryas.
2. Pushyamitra was a staunch follower of Buddhism.

Which of the statements given above is/are correct?


A. 1 only
B. 2 only
C. Both 1 and 2
D. Neither 1 nor 2

Correct Answer : A

Answer Justification :

Sungas

The founder of the Sunga dynasty was Pushyamitra Sunga, who was the
commander-in-chief under the Mauryas.

He assassinated the last Mauryan ruler and usurped the throne.

The most important challenge to the Sunga rule was to protect north India against the
invasions of the Bactrian Greeks from the northwest.

Pushyamitra was a staunch follower of Brahmanism.

He performed two asvamedha sacrifices. Buddhist sources refer him as a persecutor


of Buddhism. Hence, statement 2 is incorrect.

But there is enough evidence to show that Pushyamitra patronised Buddhist art.

During his reign the Buddhist monuments at Bharhut and Sanchi were renovated
and further improved.

After the death of Pushyamitra, his son Agnimitra became the ruler.

www.insightsactivelearn.com 2
Total Marks : 200
Test-1: GS Subject
( Insta Prelims Test Series 2022 )

The last Sunga ruler was Devabhuti, who was murdered by his minister Vasudeva
Kanva, the founder of the Kanva dynasty.

3. Consider the following statements regarding Satavahanas


1. The founder of the Satavahana dynasty was Simuka.
2. Gautamiputra Satakarni became famous for his book Gathasaptasati, also called Sattasai.
3. Nasik and Nanaghad inscriptions throw much light on the reign of Gautamiputra Satakarni.

Which of the statements given above are correct?


A. 1 and 2 only
B. 2 and 3 only
C. 1 and 3 only
D. 1, 2 and 3

Correct Answer : C

Answer Justification :

Satavahanas

The founder of the Satavahana dynasty was Simuka.

Hala became famous for his book Gathasaptasati, also called Sattasai.

It contains 700 verses in Prakrit language. Hence, statement 2 is incorrect.

In the Deccan, the Satavahanas established their independent rule after the decline of
the Mauryas.

Their rule lasted for about 450 years. They were also known as the Andhras.

The Puranas and inscriptions remain important sources for the history of Satavahanas.

Among the inscriptions, the Nasik and Nanaghad inscriptions throw much light on
the reign of Gautamiputra Satakarni.

The coins issued by the Satavahanas are also helpful in knowing the economic conditions
of that period.

www.insightsactivelearn.com 3
Total Marks : 200
Test-1: GS Subject
( Insta Prelims Test Series 2022 )

4. Consider the following statements regarding foreign invaders


1. Menander and his dialogues with the Buddhist monk Nagasena was compiled in the Pali work,
Milindapanho (Questions of Milinda).
2. A Greek ambassador Heliodorus became a Vaishnavite and erected the Garuda Pillar at
Besnagar.

Which of the statements given above is/are correct?


A. 1 only
B. 2 only
C. Both 1 and 2
D. Neither 1 nor 2

Correct Answer : C

Answer Justification :

All the above statements are correct.

Bactria and Parthia became independent from the Syrian empire in the middle of the third
century B.C. Demetrius, the Greek ruler of Bactria invaded Afghanistan and Punjab and
occupied them.

Menander was also known as Milinda and the capital of his kingdom was Sakala
(Sialcot). He evinced much interest in Buddhism and his dialogues with the Buddhist
monk Nagasena was compiled in the Pali work, Milindapanho (Questions of Milinda).
He also embraced Buddhism. A Greek ambassador Heliodorus became a Vaishnavite and
erected the Garuda Pillar at Besnagar. The Greek influence in India lasted for more than a
century after the death Menander.

5. Consider the following statements regarding Kushanas


1. The founder of the Kushana dynasty was Kujula Kadphises.
2. Kanishka was the founder of the Saka era which starts from 78 A.D.

Which of the statements given above is/are correct?


A. 1 only
B. 2 only
C. Both 1 and 2
D. Neither 1 nor 2

Correct Answer : C

Answer Justification :

www.insightsactivelearn.com 4
Total Marks : 200
Test-1: GS Subject
( Insta Prelims Test Series 2022 )

All the above statements are correct.

Kushanas

The Kushanas were a branch of Yuchi tribe, whose original home was central Asia. They
first came to Bactria displacing the Sakas. Then they gradually moved to the Kabul
valley and seized the Gandhara region.

The founder of the Kushana dynasty was Kujula Kadphises or Kadphises I.

Kanishka (78 – 120 A.D.)

Kanishka was the most important ruler of the Kushana dynasty. He was the
founder of the Saka era which starts from 78 A.D. He was not only a great
conqueror but also a patron of religion and art.

Kanishka and Buddhism

Kanishka embraced Buddhism in the early part of his reign. However, his coins exhibit
the images of not only Buddha but also Greek and Hindu gods.

6. Consider the following statements regarding Cheras


1. The Cheras ruled over parts of modern Tamil Nadu only.
2. Their important seaports were Tondi and Musiri.
3. The Pugalur inscription of the first century A.D refers to three generations of Chera rulers.

Which of the statements given above is/are correct?


A. 1 only
B. 2 and 3 only
C. 3 only
D. 1, 2 and 3

Correct Answer : B

Answer Justification :

Cheras

The Cheras ruled over parts of modern Kerala. Hence, statement 1 is incorrect.

www.insightsactivelearn.com 5
Total Marks : 200
Test-1: GS Subject
( Insta Prelims Test Series 2022 )

Their capital was Vanji and their important seaports were Tondi and Musiri.

They had the palmyra flowers as their garland.

The Pugalur inscription of the first century A.D refers to three generations of
Chera rulers.

Padirruppattu also provides information on Chera kings.

Cheran Senguttuvan belonged to 2nd century A.D.

His younger brother was Elango Adigal, the author of Silappathigaram.

Senguttuvan introduced the Pattini cult or the worship of Kannagi as the ideal wife in
Tamil Nadu.

The stone for making the idol of Kannagi was brought by him after his Himalayan
expedition.

The consecration ceremony was attended by many princes including Gajabhagu II


from Sri Lanka.

7. Consider the following statements regarding Cholas


1. Their capital was first located at Uraiyur and then shifted to Puhar.
2. Karikala was a famous king of the Sangam Cholas.
3. Padirruppattu portrays Karikala’s early life and his military conquests.

Which of the statements given above are correct?


A. 1 and 2 only
B. 2 and 3 only
C. 1 and 3 only
D. 1, 2 and 3

Correct Answer : A

Answer Justification :

Cholas

www.insightsactivelearn.com 6
Total Marks : 200
Test-1: GS Subject
( Insta Prelims Test Series 2022 )

The Chola kingdom of the Sangam period extended from modern Tiruchi district to
southern Andhra Pradesh. Their capital was first located at Uraiyur and then
shifted to Puhar. Karikala was a famous king of the Sangam Cholas.

Pattinappalai portrays his early life and his military conquests. Hence,
statement 3 is incorrect.

In the Battle of Venni he defeated the mighty confederacy consisting of the Cheras,
Pandyas and eleven minor chieftains. This event is mentioned in many Sangam poems.

Vahaipparandalai was another important battle fought by him in which nine enemy
chieftains submitted before him.

Karikala’s military achievements made him the overlord of the whole Tamil country.

Trade and commerce flourished during his reign period.

He was responsible for the reclamation of forest lands and brought them under
cultivation thus adding prosperity to the people.

He also built Kallanai across the river Kaveri and also constructed many irrigation tanks.

8. Which of the following are sources of Gupta period?


1. Devichandraguptam
2. Puranas
3. Mudhrakshasam
4. Meherauli Iron Pillar Inscription

Select the correct answer using the code given below:


A. 1, 2 and 3 only
B. 2, 3 and 4 only
C. 1, 2 and 4 only
D. 1, 2, 3 and 4

Correct Answer : D

Answer Justification :

All the above statements are correct.

www.insightsactivelearn.com 7
Total Marks : 200
Test-1: GS Subject
( Insta Prelims Test Series 2022 )

Sources

There are plenty of source materials to reconstruct the history of the Gupta period. They
include literary, epigraphical and numismatic sources.

The Puranas throw light on the royal genealogy of the Gupta kings.

Contemporary literary works like the Devichandraguptam and the Mudhrakshasam


written by Visakadatta provide information regarding the rise of the Guptas.

The Chinese traveler Fahien, who visited India during the reign of Chandragupta II, has
left a valuable account of the social, economic and religious conditions of the Gupta
empire.

Apart from these literary sources, there are inscriptions like the Meherauli Iron
Pillar Inscription and the Allahabad Pillar inscription. The first refers to the
achievements of Chandragupta I.

The most important source for the reign of Samudragupta is the Allahabad Pillar
inscription.

The coins issued by Gupta kings contain legends and figures. These coins provide
interesting details about the titles and sacrifices performed by the Gupta monarchs.

9. Consider the following statements regarding Fahien’s Visit


1. Fahien visited India during the reign of Chandragupta II.
2. According to him, Buddhism was in a degrading condition in the northwestern India.
3. He came to India by the sea route and returned by the land route.

Which of the statements given above is/are correct?


A. 1 only
B. 2 and 3 only
C. 3 only
D. 1, 2 and 3

Correct Answer : A

Answer Justification :

www.insightsactivelearn.com 8
Total Marks : 200
Test-1: GS Subject
( Insta Prelims Test Series 2022 )

Fahien’s Visit

The famous Chinese pilgrim, Fahien visited India during the reign of Chandragupta
II.

Out of his nine years stay in India, he spent six years in the Gupta empire.

He came to India by the land route through Khotan, Kashgar, Gandhara and
Punjab. He visited Peshawar, Mathura, Kanauj, Sravasti, Kapilavastu,
Kusinagara, Pataliputra, Kasi and Bodh Gaya among other places. He returned
by the sea route, visiting on the way Ceylon and Java. Hence, statement 3 is
incorrect.

The main purpose of his visit was to see the land of the Buddha and to collect Buddhist
manuscripts from India. He stayed in Pataliputra for three years studying Sanskrit and
copying Buddhist texts.

Fahien provides valuable information on the religious, social and economic condition of
the Gupta empire. According to him, Buddhism was in a flourishing condition in
the northwestern India but in the Gangetic valley it was in a state of neglect. He refers
to the Gangetic valley as the ‘land of Brahmanism’. Hence, statement 2 is incorrect.

10. Consider the following statements regarding Social Life under Guptas
1. During the Gupta period, the caste system became rigid.
2. The practice of Swyamvara was given up.
3. Women were permitted to studying the religious texts like the Puranas.

Which of the statements given above are correct?


A. 1 and 2 only
B. 2 and 3 only
C. 1 and 3 only
D. 1, 2 and 3

Correct Answer : A

Answer Justification :

Social Life

The pre-Gupta period in India witnessed a series of foreign invasions. Indian society had

www.insightsactivelearn.com 9
Total Marks : 200
Test-1: GS Subject
( Insta Prelims Test Series 2022 )

given way to those foreigners who had become permanent residents here. But during
the Gupta period, the caste system became rigid.

The position of women had also become miserable during the Gupta period.

They were prohibited from studying the religious texts like the Puranas.
Hence, statement 3 is incorrect.

The subjection of women to men was thoroughly regularized. But it was insisted
that they should be protected and generously treated by men.

The practice of Swyamvara was given up and the Manusmriti suggested the
early marriage for girls.

11. Which of the following battle ended with the Treaty of Allahabad?

A. Revolt of 1857
B. Battle of Buxar
C. First Anglo-Mysore War
D. Second Carnatic war

Correct Answer : B

Answer Justification :

The Treaty of Allahabad was signed on August 16, 1765 between Mughal Emperor Shah
Alam II of Bengal and Lord Clive of the British East India Company after the Battle of
Buxar (1764). Based on the terms of the agreement, Shah Alam II granted Diwani rights to the
East India Company.

12. Consider the following pairs of Astapradhan under Shivaji


administration and their role:
Ashtapradhans: Role:
1. Sumanta – Military commander
2. Amatya – Accountant General
3. Sar-i-Naubat – Master of ceremonies
4. Waqenavis – Intelligence, posts and household affairs

www.insightsactivelearn.com 10
Total Marks : 200
Test-1: GS Subject
( Insta Prelims Test Series 2022 )

Which of the statement above is/are correct?


A. 1 and 2 only
B. 2 and 4 only
C. 3 and 4 only
D. 1 and 4 only

Correct Answer : B

Answer Justification :

Shivaji was also a great administrator. He laid the foundations of a sound system of
administration. The king was the pivot of the government. He was assisted by a council
of ministers called Ashtapradhan. However, each minister was directly responsible to
Shivaji.

1. Peshwa – Finance and general administration. Later he became the prime minister.

2. Sar-i-Naubat or Senapati – Military commander, an honorary post.

3. Amatya – Accountant General.

4. Waqenavis – Intelligence, posts and household affairs.

5. Sachiv – Correspondence.

6. Sumanta – Master of ceremonies.

7. Nyayadish – Justice.

8. Panditarao – Charities and religious administration.

13. Consider the following statements


1. The Chauth and Sardeshmukhi were taxes conceived during the reign of Balaji Bajirao
2. The Sardeshmuki directly went to the King's treasury

Which of the statements given above is/are correct?


A. 1 only
B. 2 only
C. Both 1 and 2
D. Neither 1 nor 2

Correct Answer : B

Answer Justification :

www.insightsactivelearn.com 11
Total Marks : 200
Test-1: GS Subject
( Insta Prelims Test Series 2022 )

Chauth and Sardeshmukhi were taxes conceived during the times of the Great
Maratha Ruler Shivaji Maharaj.

Hence, statement 1 is incorrect.

'Chauth' means basically 1/4th i.e 25% of gross revenue or produce to be paid to
jagirdars of Maratha Empire from hostile or alien state. In return the state paying the
tax would get an assuarance of non-aggression from the Maratha army against the enemy
state.

'Sardeshmukhi' is an additional 10% tax levied upon the collected 'Chauth'. The reason for the
additional tax was due to the King claiming hereditary rights upon the tax collection.

The Sardeshmuki directly went to the King's treasury whereas the share of king in
Chauth was 1/3rd of the gross Chauth collected.

Hence, statement 2 is correct.

14. The famous book Akbar Nama was written by

A. Abu’l Fazl
B. Shah Jahan
C. Jahangir
D. Abdul Hamid Lahori

Correct Answer : A

Answer Justification :

15. Consider the following statements regarding China and Central & Eastern European (CEE) 17+1
1. The 17+1 initiative is a China-led format founded in 2012 in Beijing.

www.insightsactivelearn.com 12
Total Marks : 200
Test-1: GS Subject
( Insta Prelims Test Series 2022 )

2. Lithuania recently joined this group.


3. Bosnia and Serbia are the two Non-European Union members of this group

Which of the statements given above is/are correct?


A. 1 only
B. 1 and 3 only
C. 3 only
D. 2 and 3 only

Correct Answer : C

Answer Justification :

China and Central & Eastern European (CEE) 17+1 mechanism:

Context:

Lithuania has justified it’s decision to pull out of the China and Central & Eastern European
(CEE) 17+1 mechanism, which is seen as a pro-China grouping of countries within the EU. It
has also denied that the decision came because of U.S. pressure. Hence Statement 2 is
incorrect.

What was the main trigger for this decision?

1. The ‘17 plus one’ format became a divisive forum; it was supposed to act as Europe’s one
strong voice.

2. Tensions between China and Lithuania have been building up over a number of
issues: Lithuania’s new ties with Taiwan, its Parliament’s resolution on Uighurs, and then
Chinese sanctions on Lithuanian and EU politicians.

3. China has also decided to impose entry bans and sanctions against some European
politicians and academics, and that has had an impact on EU-China relations.

What is the “17+1” initiative?

The 17+1 initiative is a China-led format founded in 2012 in Budapest with an aim to expand
cooperation between Beijing and the Central and Eastern European (CEE) member
countries, with investments and trade for the development of the CEE region. Hence
Statement 1 is incorrect.

The framework also focuses on infrastructure projects such as bridges, motorways,


railway lines and modernisation of ports in the member states.

www.insightsactivelearn.com 13
Total Marks : 200
Test-1: GS Subject
( Insta Prelims Test Series 2022 )

What’s in it for China?

China’s narrative towards the 17+1 initiative is about improving its relations with the
European countries that are less developed as compared to the Western European states.

However, the platform is largely seen as an extension of China’s flagship Belt and
Road initiative (BRI).

Composition:

The initiative includes twelve EU member states and five Balkan states — Albania, Bosnia and
Herzegovina, Bulgaria, Croatia, Czech Republic, Estonia, Greece, Hungary, Latvia, Lithuania,
Macedonia, Montenegro, Poland, Romania, Serbia, Slovakia and Slovenia. Hence Statement
3 is correct.

https://www.insightsonindia.com/2021/06/02/china-and-central-eastern-european-cee-171-mec
hanism/

16. Consider the following statements regarding National Commission for Protection of Child Rights (NCPCR)
1. It is a statutory body
2. It works under the aegis of Ministry of Women and Child development
3. Under the RTE Act, 2009, the NCPCR can inquire into complaints about violation of the law.

Which of the statements given above is/are correct?


A. 1 and 2 only
B. 2 only
C. 3 only
D. 1, 2 and 3

www.insightsactivelearn.com 14
Total Marks : 200
Test-1: GS Subject
( Insta Prelims Test Series 2022 )

Correct Answer : D

Answer Justification :

The National Commission for Protection of Child Rights (NCPCR) is an Indian


governmental commission, established by an Act of Parliament, the Commission for Protection
of Child Rights Act in December 2005, thus is a statutory body. Hence, statement 1 is
correct.

The commission works under the aegis of Ministry of Women and Child development,
GoI. The Commission began operation a year later in March 2007. Hence, statement 2 is
correct.

The Commission considers that its Mandate is "to ensure that all Laws, Policies, Programmes,
and Administrative Mechanisms are in consonance with the Child Rights perspective as
enshrined in the Constitution of India and the UN Convention on the Rights of the Child."

Under the RTE Act, 2009, the NCPCR can:

1. inquire into complaints about violation of the law.

2. summon an individual and demand evidence.

3. seek a magisterial enquiry.

4. file a writ petition in the High Court or Supreme Court.

5. approach the government concerned for prosecution of the offender.

6. recommend interim relief to those affected. Hence Statement 3 is correct.

Composition:

This commission has a chairperson and six members of which at least two should be women.

1. All of them are appointed by Central Government for three years.

2. The maximum age to serve in commission is 65 years for Chairman and 60 years for
members.

www.insightsactivelearn.com 15
Total Marks : 200
Test-1: GS Subject
( Insta Prelims Test Series 2022 )

https://www.insightsonindia.com/2021/06/02/national-commission-for-protection-of-child-rights
-ncpcr-2/

17. Bhashan Char Island, recently seen in news, is located in?

A. Gulf of Mannar
B. South China Sea
C. Gulf of Oman
D. Bay of Bengal

Correct Answer : D

Answer Justification :

Thousands of Rohingya protest at Bhashan Char:

Context:

Several thousand Rohingya recently staged “unruly” protests against living conditions
on Bhashan Char- a cyclone-prone island off Bangladesh.

What’s the issue?

Since December, Bangladesh has shifted 18,000 refugees to the low-lying silt island of
Bhashan Char from the mainland Bangladesh, where around 8,50,000 people live in squalid
and cramped conditions.

What’s the main concern now?

Bhasan Char (Floating Island) also known as Char Piya or Thengar Char Island, is an
island in Hatiya, Bangladesh.

The island was formed from a build-up of silt in the Bay of Bengal only 20 years ago, and
concerns have been consistently raised about Bhasan Char’s exposure to extreme
weather and distance from the mainland in emergencies.

www.insightsactivelearn.com 16
Total Marks : 200
Test-1: GS Subject
( Insta Prelims Test Series 2022 )

https://www.insightsonindia.com/2021/06/01/thousands-of-rohingya-protest-at-bhashan-char/

18. Consider the following statements regarding Digital Tax in India


1. The equalization levy was imposed to give level playing field between Indian businesses who
pay tax in India and foreign e-commerce companies who do business in India but do not pay
any income tax in India
2. Offshore e-commerce firms that sell through an Indian arm will not have to pay equalization
levy.

Which of the statements given above is/are correct?


A. 1 only
B. 2 only
C. Both 1 and 2
D. Neither 1 nor 2

www.insightsactivelearn.com 17
Total Marks : 200
Test-1: GS Subject
( Insta Prelims Test Series 2022 )

Correct Answer : C

Answer Justification :

Both the statements are correct.

Digital tax in India:

Context:

The United States recently announced 25% tariffs on over $2 billion worth of imports
from six nations over their digital services taxes, but immediately suspended the duties to
allow time for international tax negotiations to continue.

The US. Trade Representative’s office had approved the threatened tariffs on goods
from Britain, Italy, Spain, Turkey, India and Austria after a “Section
301” investigation concluded that their digital taxes discriminated against U.S.
companies.

The potential tariffs aim to equal the amount of digital taxes that would be
collected from U.S. firms.

About the Digital Tax:

India was the one of the first countries to introduce a 6 per cent equalisation levy in
2016, but the levy was restricted to online advertisement services.

However, India introduced the digital tax in April 2020 for foreign companies
selling goods and services online to customers in India and showing annual revenues more
than INR 20 million.

Applicability:

India has expanded the scope of the equalisation levy over the last few years, to tax non-
resident digital entities.

While the levy applied only to digital advertising services till 2019-20 at the rate
of 6 percent, the government in April last year widened the scope to impose a 2 per
cent tax on non-resident e-commerce players with a turnover of Rs 2 crore.

The scope was further widened in the Finance Act 2021-22 to cover e-commerce
supply or service when any activity takes place online.

www.insightsactivelearn.com 18
Total Marks : 200
Test-1: GS Subject
( Insta Prelims Test Series 2022 )

Since May 2021, this also includes any entity that systematically and continuously does
business with more than 3 lakh users in India.

When will the tax not apply?

Offshore e-commerce firms that sell through an Indian arm will not have to pay.

This means if the goods and services sold on a foreign e-commerce platform are owned
or provided by an Indian resident or Indian permanent establishment, they will not be
subject to the two percent equalization levy.

Why was it imposed?

The equalisation levy was imposed “to give level playing field between Indian
businesses who pay tax in India and foreign e-commerce companies who do business
in India but do not pay any income tax here.

Which other countries impose such a levy on digital sellers?

1. France imposes a three percent digital services tax.

2. In the ASEAN region, Singapore, Indonesia, and Malaysia impose a digital service tax
with Thailand announcing forthcoming plans to tax its foreign digital service providers.

3. Negotiations are underway at the Organisation for Economic Cooperation and


Development (OECD) involving 140 countries to overhaul international tax rules given
the fast growth of internet economies.

Why the United States Trade Representative (USTR) says that this tax is
discriminatory?

1. First, it states that the DST discriminates against US digital businesses because it
specifically excludes from its ambit domestic (Indian) digital businesses.

2. USTR also says the DST is discriminatory because it does not extend to identical services
provided by non-digital service providers.

Why India says Digital services tax is not discriminatory? And why is it needed?

Business models employed by non-resident digital service providers obviate the

www.insightsactivelearn.com 19
Total Marks : 200
Test-1: GS Subject
( Insta Prelims Test Series 2022 )

need for a physical presence in India and profits earned here could easily escape the
Indian income tax net. Hence, this kind of taxation is necessary.

Changing International Economic Order: Countries such as India which provide


large markets for digital corporations seek a greater right to tax incomes.

Associated Concerns:

1. Eventually the tax may become a burden for Digital Consumers.

2. It could invite retaliatory tariffs (such as the latest one), as similar tariffs were
imposed by the US on France.

3. It would also result in double taxation.

https://www.insightsonindia.com/2021/06/03/digital-tax-in-india/

19. Davinci+, Veritas missions, recently announced by NASA is related to which planet?

A. Venus
B. Mercury
C. Mars
D. Asteroid Belt

Correct Answer : A

Answer Justification :

NASA announces two new missions to Venus:

Context:

The last US probe to visit the planet was the Magellan orbiter in 1990.

Now, NASA has announced two new missions to Venus. These two sister missions both aim to
understand how Venus became an inferno-like world, capable of melting lead at the
surface. These include:

1. Davinci+:

The Davinci+ (Deep Atmosphere Venus Investigation of Noble gases, Chemistry, and

www.insightsactivelearn.com 20
Total Marks : 200
Test-1: GS Subject
( Insta Prelims Test Series 2022 )

Imaging) mission will:

Measure the planet’s atmosphere to gain insight into how it formed and evolved.

Determine whether Venus ever had an ocean.

Return the first high resolution images of the planet’s “tesserae” geological features
(These features could be comparable to continents on Earth).

2. Veritas (Venus Emissivity, Radio Science, InSAR, Topography, and Spectroscopy):

This mission will map the planet’s surface to understand its geological history and investigate
how it developed so differently than Earth.

It will use a form of radar to chart surface elevations and discover whether volcanoes
and earthquakes are still happening.

About Venus:

Venus is the second planet from the sun and the hottest planet in the solar
system with a surface temperature of 500C – high enough to melt lead.

The planet’s thick atmosphere has cranked the surface pressure up to 90 bars.

A single Venusian rotation takes 243.0226 Earth days. That means a day lasts
longer than a year on Venus, which makes a complete orbit around the sun in 225
Earth days.

The Venusian planetary core has a diameter of about 4,360 miles (7,000 km),
comparable to Earth’s core.

Venus is one of just two planets that rotate from east to west. Only Venus and Uranus
have this “backwards” rotation.

Historic missions to Venus:

1. Magellan – a Nasa mission that ended in 1994.

www.insightsactivelearn.com 21
Total Marks : 200
Test-1: GS Subject
( Insta Prelims Test Series 2022 )

2. Venus Express– A European mission- focused on atmospheric science.

3. Akatsuki– Japanese spacecraft- focused on atmospheric science.

Future missions:

The European Space Agency (Esa) is evaluating a Venus mission, called EnVision, alongside
two astronomy proposals – Theseus and Spica. Other concepts are also being proposed to
Nasa.

https://www.insightsonindia.com/2021/06/04/nasa-announces-two-new-missions-to-venus/

20. Consider the following statements regarding Nagorno-Karabakh region


1. It is a region located in West Asia
2. Majority of population residing in this region are Afghans

Which of the statements given above is/are correct?


A. 1 only
B. 2 only
C. Both 1 and 2
D. Neither 1 nor 2

Correct Answer : D

Answer Justification :

Both the statements are incorrect.

Nagorno-Karabakh region:

Nagorno-Karabakh is part of Azerbaijan, but its population is majority Armenian. As the


Soviet Union saw increasing tensions in its constituent republics in the 1980s, Nagorno-
Karabakh voted to become part of Armenia – sparking a war which stopped with a
ceasefire in 1994.

Since then, Nagorno-Karabakh has remained part of Azerbaijan but is controlled


by separatist ethnic Armenians backed by the Armenian government.

Context:

A brief war took place last year in a region recaptured in disputed Nagorno-Karabakh.

www.insightsactivelearn.com 22
Total Marks : 200
Test-1: GS Subject
( Insta Prelims Test Series 2022 )

Azerbaijan seized back swathes of territory in the separatist ethnic Armenia region
with the help of Turkish combat drones and other weaponry from Ankara.

The conflict claimed 6,000 lives and ended with a Russian-brokered truce in
November that saw ethnic Armenians pushed out of large parts of the land they had won
during a brutal post-Soviet war in the 1990s.

https://www.insightsonindia.com/2021/06/10/insights-daily-current-affairs-pib-summary-10-jun
e-2021/

21. Consider the following statements regarding Mahendravarman I


1. He was also known as Mamalla, which means ‘great wrestler’.
2. His victory over Pulakesin II in the Battle of Manimangalam near Kanchi is mentioned in
Kuram copper plates.
3. He was converted to Saivism by the influence of the Saiva saint, Appar.

Which of the statements given above is/are correct?


A. 1 only

www.insightsactivelearn.com 23
Total Marks : 200
Test-1: GS Subject
( Insta Prelims Test Series 2022 )

B. 2 and 3 only
C. 3 only
D. 1, 2 and 3

Correct Answer : C

Answer Justification :

Mahendravarman I (600 – 630 A.D.)

The long-drawn Pallava – Chalukya Conflict began during his period. Pulakesin II
marched against the Pallavas and captured the northern part of their kingdom. Although
a Pallava inscription refers to the victory of Mahendravarman I at Pullalur, he was not
able to recover the lost territory.

Mahendravarman I was a follower of Jainism in the early part of his career.

He was converted to Saivism by the influence of the Saiva saint,


Thirunavukkarasar alias Appar.

The Mandagappattu inscription hails him as Vichitrachitta who constructed a temple for
Brahma, Vishnu and Siva without the use of bricks, timber, metal and mortar.

He is also regarded as an expert in music. The music inscription at Kudumianmalai is


ascribed to him.

Narasimhavarman I (630-668 A.D.)

Narasimhavarman I was also known as Mamalla, which means ‘great wrestler’. He


wanted to take avenge the defeat of his father at the hands of Chalukyan ruler Pulakesin II.
His victory over Pulakesin II in the Battle of Manimangalam near Kanchi is
mentioned in Kuram copper plates.

Hence, both statement 1 and 2 are incorrect.

22. Consider the following statements regarding Vedic literature


1. The term ‘Veda’ signifies ‘superior knowledge’.
2. The Atharva Veda consists of various details of rules to be observed at the time of sacrifice.
3. The Brahmanas are the treatises relating to prayer and sacrificial ceremony.
www.insightsactivelearn.com 24
Total Marks : 200
Test-1: GS Subject
( Insta Prelims Test Series 2022 )

Which of the statements given above are correct?


A. 1 and 2 only
B. 2 and 3 only
C. 1 and 3 only
D. 1, 2 and 3

Correct Answer : C

Answer Justification :

The word ‘Veda’ is derived from the root ‘vid’, which means to know. In other words, the term
‘Veda’ signifies ‘superior knowledge’.

The Vedic literature consists of the four Vedas – Rig, Yajur, Sama and Atharva.

The Rig Veda is the earliest of the four Vedas and it consists of 1028 hymns. The hymns
were sung in praise of various gods.

The Yajur Veda consists of various details of rules to be observed at the time of
sacrifice. Hence, statement 2 is incorrect.

The Sama Veda is set to tune for the purpose of chanting during sacrifice. It is called the
book of chants and the origins of Indian music are traced in it.

The Atharva Veda contains details of rituals.

Besides the Vedas, there are other sacred works like the Brahmanas, the Upanishads,
the Aranyakas and the epics Ramayana and Mahabharata.

The Brahmanas are the treatises relating to prayer and sacrificial ceremony.

The Upanishads are philosophical texts dealing with topic like the soul, the absolute, the
origin of the world and the mysteries of nature.

www.insightsactivelearn.com 25
Total Marks : 200
Test-1: GS Subject
( Insta Prelims Test Series 2022 )

The Aranyakas are called forest books and they deal with mysticism, rites, rituals and
sacrifices.

23. Consider the following statements regarding Aryan culture


1. The principal traits of Aryan culture are set out by Vedic, Iranian, and Greek literary texts.
2. Generally, the texts represent agriculture and pastoralism as the principal sources of
livelihood.
3. They used spoked wheels, and fought with bows and arrows which were placed in quivers.

Which of the statements given above is/are correct?


A. 1 only
B. 2 and 3 only
C. 3 only
D. 1, 2 and 3

Correct Answer : D

Answer Justification :

All the above statements are correct.

The principal traits of Aryan culture are set out by Vedic, Iranian, and Greek
literary texts and cognate terms found in the proto-Indo-European languages.

The texts that help us to reconstruct the material and other aspects of Aryan culture
comprise the Rig Veda, the Zend-Avesta, and Homer’s Iliad and Odyssey. Specialists may
differ on the criteria for dating these texts, but we may go by the generally accepted
dates.

The Rig Veda is assigned to roughly 1500 BC, although the later additions might be as
late as 1000 BC.

The earliest parts of the Zend- Avesta are roughly attributed to 1400 BC, and Homer’s
works are assigned to 900–800 BC.

Though these texts belong to different areas, they suggest the period when copper and
bronze were in use.

The later portions of Homer also mention iron. Generally, the texts represent
agriculture and pastoralism as the principal sources of livelihood.

www.insightsactivelearn.com 26
Total Marks : 200
Test-1: GS Subject
( Insta Prelims Test Series 2022 )

The people lived in temperate climate.

They domesticated horses which were used for riding and for driving carts.

They used spoked wheels, and fought with bows and arrows which were placed
in quivers.

They lived in a male-dominated society.

They buried the dead body, but also practiced cremation.

The cults of fire and soma prevailed among the speakers of the Indo-European languages
in Iran and the Indian subcontinent.

However, animal sacrifice, including horse sacrifice, seems to have been practiced by all
the Indo-European communities.

24. Consider the following statements regarding Nalanda University


1. Hiuen Tsang gives a very valuable account of the Nalanda University.
2. The medium of instruction was Prakrit.
3. It was a Hinayana University

Which of the statements given above is/are correct?


A. 1 only
B. 2 and 3 only
C. 3 only
D. 1, 2 and 3

Correct Answer : A

Answer Justification :

Nalanda University

The Chinese travelers of ancient India mentioned a number of educational institutions.


The most famous among them were the Hinayana University of Valabhi and the
Mahayana University of Nalanda. Hence, statement 3 is incorrect.

Hiuen Tsang gives a very valuable account of the Nalanda University.

www.insightsactivelearn.com 27
Total Marks : 200
Test-1: GS Subject
( Insta Prelims Test Series 2022 )

The term Nalanda means “giver of knowledge”.

It was founded by Kumaragupta I during the Gupta period.

It was patronised by his successors and later by Harsha.

The professors of the University were called panditas.

Some of its renowned professors were Dingnaga, Dharmapala, Sthiramati and


Silabadhra. Dharmapala was a native of Kanchipuram and he became the head of the
Nalanda University.

Though it was a Mahayana University, different religious subjects like the


Vedas, Hinayana doctrine, Sankhya and Yoga philosophies were also taught.

In addition to that, general subjects like logic, grammar, astronomy, medicine and art
were in the syllabus. Discipline was very strict.

More than lectures, discussion played an important part and the medium of
instruction was Sanskrit. Hence, statement 2 is incorrect.

25. Consider the following statements regarding Indo-Aryan conflicts


1. Aryans were divided into five tribes called panchajana.
2. The Bharatas and the Purus were the ruling Aryan clans, and they were supported by priest
Vasishtha.
3. Battle of Ten Kings was fought on the river Parushni.

Which of the statements given above are correct?


A. 1 and 2 only
B. 2 and 3 only
C. 1 and 3 only
D. 1, 2 and 3

Correct Answer : C

Answer Justification :

www.insightsactivelearn.com 28
Total Marks : 200
Test-1: GS Subject
( Insta Prelims Test Series 2022 )

The Indo-Aryans were engaged in two types of conflicts. First, they fought with the pre-
Aryans, and secondly, they fought amongst themselves.

Intra tribal conflicts rocked the Aryan communities for a long time. According to
tradition, the Aryans were divided into five tribes called panchajana, but there might
have been other tribes too.

The Aryans fought amongst themselves and sometimes enlisted the support of the non-
Aryan peoples for the purpose.

The Bharatas and the Tritsu were the ruling Aryan clans, and they were
supported by priest Vasishtha. Hence, statement 2 is incorrect.

The country Bharatavarsha was eventually named after the tribe Bharata, which is first
mentioned in the Rig Veda.

The Bharata ruling clan was opposed by a host of ten chiefs, five of whom were heads of
Aryan tribes and the remaining five of the non-Aryan people.

The battle that the Bharatas fought with the host of ten chiefs is known as the Battle of
Ten Kings.

It was fought on the river Parushni, coterminous with the river Ravi, and it gave victory
to Sudas and established the supremacy of the Bharatas.

Of the defeated tribes, the most important was that of the Purus.

26. Consider the following statements regarding Later Vedic Period


1. Satapatha Brahmana refers to the expansion of Aryans to the eastern Gangetic plains.
2. Pravahana Jaivali’s court, the king of Videha, was adorned by scholar Yajnavalkya.

Which of the statements given above is/are correct?


A. 1 only
B. 2 only
C. Both 1 and 2
D. Neither 1 nor 2

Correct Answer : A

www.insightsactivelearn.com 29
Total Marks : 200
Test-1: GS Subject
( Insta Prelims Test Series 2022 )

Answer Justification :

Later Vedic Period (1000 – 600 B.C.)

The Aryans further moved towards east in the Later Vedic Period. The Satapatha
Brahmana refers to the expansion of Aryans to the eastern Gangetic plains.

Kuru and Panchala kingdoms flourished in the beginning. Parikshat and Janamejaya
were the famous rulers of Kuru kingdom. Pravahana Jaivali was a popular king of the
Panchalas. He was a patron of learning. After the fall of Kurus and Panchalas, other
kingdoms like Kosala, Kasi and Videha came into prominence. The famous ruler of Kasi
was Ajatasatru.

Janaka was the king of Videha with its capital at Mithila. His court was adorned
by scholar Yajnavalkya. Hence, statement 2 is incorrect.

Magadha, Anga and Vanga seem to be the easternmost tribal kingdoms.

The later Vedic texts also refer to the three divisions of India – Aryavarta (northern
India), Madhyadesa (central India) and Dakshinapatha (southern India).

27. Consider the following statements regarding Bimbisara


1. Bimbisara belonged to the Haryanka dynasty.
2. Bimbisara was a contemporary of both Vardhamana Mahavira and Gautama Buddha.
3. He was instrumental in convening the First Buddhist Council at Rajagriha soon after the death
of the Buddha.

Which of the statements given above are correct?


A. 1 and 2 only
B. 2 and 3 only
C. 1 and 3 only
D. 1, 2 and 3

Correct Answer : A

Answer Justification :

Bimbisara (546 - 494 B.C.)

Bimbisara belonged to the Haryanka dynasty.

www.insightsactivelearn.com 30
Total Marks : 200
Test-1: GS Subject
( Insta Prelims Test Series 2022 )

He consolidated his position by matrimonial alliances. His first matrimonial alliance was
with the ruling family of Kosala.

Bimbisara also undertook many expeditions and added more territories to his empire. He
defeated Brahmadatta of Anga and annexed that kingdom. He maintained friendly
relations with Avanti. He had also efficiently reorganized the administration of his
kingdom.

Bimbisara was a contemporary of both Vardhamana Mahavira and Gautama


Buddha. However, both religions claim him as their supporter and devotee. He seems to
have made numerous gifts to the Buddhist Sangha.

Ajatasatru

According to the Mahavamsa, he constructed several chaityas and viharas. He was also
instrumental in convening the First Buddhist Council at Rajagriha soon after the
death of the Buddha. Hence, statement 3 is incorrect.

28. Consider the following statements regarding Nandas


1. Dhana Nanda was the founder of Nanda Kingdom.
2. The Hathigumpha inscription of Kharavela of Kalinga refers to the conquest of Kalinga by the
Nandas.
3. The Puranas speak of the extensive conquests made by Mahapadma Nanda.

Which of the statements given above are correct?


A. 1 and 2 only
B. 2 and 3 only
C. 1 and 3 only
D. 1, 2 and 3

Correct Answer : B

Answer Justification :

Nandas

Mahapadma Nanda was a powerful ruler of the Nanda dynasty. He uprooted the
kshatriya dynasties in north India and assumed the title ekarat.

The Puranas speak of the extensive conquests made by Mahapadma. The


Hathigumpha inscription of Kharavela of Kalinga refers to the conquest of

www.insightsactivelearn.com 31
Total Marks : 200
Test-1: GS Subject
( Insta Prelims Test Series 2022 )

Kalinga by the Nandas.

Many historians believe that a considerable portion of the Deccan was also under
the control of the Nandas. Therefore, Mahapadma Nanda may be regarded as a
great empire builder.

According to the Buddhist tradition, Mahapadma Nanda ruled about ten years. He was
succeeded by his eight sons, who ruled successively.

The last Nanda ruler was Dhana Nanda. Hence, statement 1 is incorrect.

He kept the Magadhan empire intact and possessed a powerful army and enormous
wealth. The fabulous wealth of the Nandas is also mentioned by several sources.

The enormous wealth of the Nandas is also referred to in the Tamil Sangam
work Ahananuru by the poet Mamulanar.

29. Consider the following statements regarding Cyrus


1. He was the first conqueror who led an expedition and entered into India.
2. He fought the famous battle of Hydaspes against Porus.

Which of the statements given above is/are correct?


A. 1 only
B. 2 only
C. Both 1 and 2
D. Neither 1 nor 2

Correct Answer : A

Answer Justification :

Cyrus the Great was the greatest conqueror of the Achaemenian Empire. He was the first
conqueror who led an expedition and entered into India. He captured the Gandhara
region.

Battle of Hydaspes

In 327 B.C. Alexander crossed the Hindukush Mountains and spent nearly ten months in

www.insightsactivelearn.com 32
Total Marks : 200
Test-1: GS Subject
( Insta Prelims Test Series 2022 )

fighting with the tribes. He crossed the Indus in February 326 B.C. with the help of the bridge
of boats. Then Alexander marched from Taxila to the banks of the river Hydaspes (Jhelum). On
the other side of the river he saw the vast army of Porus. After a few days, he crossed the
river and the famous battle of Hydaspes was fought on the plains of Karri. It was a
well-contested battle. Although Porus had a strong army, he lost the battle. Alexander was
impressed by the courage and heroism of this Indian prince, treated him generously and
reinstated him on his throne. Hence, statement 2 is incorrect.

30. Consider the following statements regarding Kautilya


1. Kautilya was called ‘Indian Machiavelli’.
2. The manuscript of Arthasastra was first discovered by R. Shama Sastri in 1904.
3. The Arthasastra contains 15 books and 180 chapters.

Which of the statements given above are correct?


A. 1 and 2 only
B. 2 and 3 only
C. 1 and 3 only
D. 1, 2 and 3

Correct Answer : D

Answer Justification :

All the above statements are correct.

Kautilya’s Arthasastra

This book in Sanskrit was written by Kautilya, a contemporary of Chandragupta Maurya.


Kautilya was also called ‘Indian Machiavelli’. The manuscript of Arthasastra was first
discovered by R. Shama Sastri in 1904. The Arthasastra contains 15 books and 180
chapters but it can be divided into three parts: the first deals with the king and his
council and the departments of government; the second with civil and criminal law; and the
third with diplomacy and war. It is the most important literary source for the history of the
Mauryas.

31. Consider the following statements regarding UN Security Council.


1. UNSC recommends the admission of new Members to UNGA.
2. The UN Charter doesn’t clearly define the functions and powers of the Secretary General
3. More than 60 United Nations Member States have never been Members of the Security
Council.
4. A State which is a Member of the United Nations but not of the Security Council may
participate, with right to vote, in its discussions when the Council considers that country's
interests are affected.

www.insightsactivelearn.com 33
Total Marks : 200
Test-1: GS Subject
( Insta Prelims Test Series 2022 )

Which of the statements given above is/are correct?


A. 1 and 3 only
B. 1 and 4 only
C. 2, 3 and 4 only
D. 1, 2 and 3 only

Correct Answer : D

Answer Justification :

UN Security Council - UNSC

UNSC recommends the admission of new Members to UNGA. Hence Statement 1 is


correct.

More than 60 United Nations Member States have never been Members of the Security
Council. Hence Statement 3 is correct.

A State which is a Member of the United Nations but not of the Security Council may
participate, without a vote, in its discussions when the Council considers that country's
interests are affected. Hence Statement 4 is incorrect.

Both Members and non-members of the United Nations, if they are parties to a dispute being
considered by the Council, may be invited to take part, without a vote, in the Council's
discussions; the Council sets the conditions for participation by a non-member State.

Issues/challenges with the office of UN Secretary General:

1. The UN Charter doesn’t clearly define the functions and powers of the Secretary
General. Hence Statement 2 is correct.

2. Selection is not done entirely on merit and transparency.

3. Critics of the appointment process say it lacks transparency and falls prey to cronyism
due to the permanent Security Council members’ veto power and their secret
negotiations over candidates.

4. The secretary-general often struggles to balance the interests of other large funders and
powerful member states as well.

Significance of the office:

www.insightsactivelearn.com 34
Total Marks : 200
Test-1: GS Subject
( Insta Prelims Test Series 2022 )

1. Peacekeeping: The secretary-general’s office oversees peacekeeping missions and


appoints the undersecretary in charge of that department.

2. Mediation: As part of the “good offices” responsibility of the position, the secretary-
general practices independence and impartiality to prevent and limit conflict.

https://www.un.org/securitycouncil/content/current-members

https://www.insightsonindia.com/2021/06/09/un-security-council-endorses-secretary-general-g
uterres-for-second-term/

32. Consider the following statements


1. In case of non-supply of entitled food grains or meals, there is a provision for food security
allowance to entitled beneficiaries.
2. Every pregnant and lactating mother is entitled to a free meal at the local anganwadi as well
as maternity benefits of Rs.60,000 in instalments under NFSA, 2013
3. NFSA also guarantees age appropriate meal, free of charge through local anganwadi for
children up to 6 months and one free meal for children in age group 6-14 years in schools

Which of the statements given above is/are correct?


A. 1 and 3 only
B. 2 and 3 only
C. 1 and 2 only
D. 1, 2 and 3

Correct Answer : A

Answer Justification :

National Food Security Act (NFSA):

Context:

The Centre has informed the Supreme Court that the States and the Union Territories have
purchased nearly 3.7 lakh tonnes of foodgrains at concessional rates from the Food
Corporation of India this year to supply to migrants without ration cards and others outside
the protective cover of the National Food Security Act (NFSA).

With this, the centre has debunked apprehensions raised in court that those without
ration cards may be left to die amid a devastating pandemic.

Background:

www.insightsactivelearn.com 35
Total Marks : 200
Test-1: GS Subject
( Insta Prelims Test Series 2022 )

The Supreme Court had asked the Centre to explain “how food will reach migrant labourers
without ration cards”.

National Food Security Act (NFSA), 2013:

The objective is to provide for food and nutritional security in human life cycle approach, by
ensuring access to adequate quantity of quality food at affordable prices to people to live a life
with dignity.

Key features:

Coverage and entitlement under Targeted Public Distribution System (TPDS): The
TDPS covers 50% of the urban population and 75% of the rural population, with uniform
entitlement of 5 kg per person per month. However, the poorest of the poor households will
continue to receive 35 kg of food grains per household per month under Antyodaya Anna
Yojana (AAY).

Subsidised prices under TPDS and their revision: For a period of three years from the
date of commencement of the Act, Food grains under TPDS will be made available at
subsidised prices of Rs. 3/2/1 per kg for rice, wheat and coarse grains.

Identification of Households: The identification of eligible households is to be done by


States/UTs under TDPS determined for each State.

Nutritional Support to women and children: Children in the age group of 6 months to 14
years and pregnant women and lactating mothers will be entitled to meals as per prescribed
nutritional norms under Integrated Child Development Services (ICDS) and Mid-Day Meal
(MDM) schemes. Malnourished children up to the age of 6 have been prescribed for higher
nutritional norms.

Maternity Benefit: Pregnant women and lactating mothers will also be receiving
maternity benefit of Rs. 6,000. Hence Statement 2 is incorrect.

Women Empowerment: For the purpose of issuing of ration cards, eldest woman of the
household of age 18 years or above is to be the head of the household.

Grievance Redressal Mechanism: Grievance redressal mechanism available at the District


and State levels.

Cost of transportation & handling of food grains and Fair Price Shop (FPS) Dealers’
margin: the expenditure incurred by the state on transportation of food grains within the
State, its handling and FPS dealers’ margin as per norms to be devised for this purpose and
assistance to states will be provided by the Central Government to meet the above
expenditure.

Transparency and Accountability: In order to ensure transparency and accountability,


provisions have been made for disclosure of records relating to PDS, social audits and
setting up of Vigilance Committees.

www.insightsactivelearn.com 36
Total Marks : 200
Test-1: GS Subject
( Insta Prelims Test Series 2022 )

Food Security Allowance: In case of non-supply of entitled food grains or meals, there is a
provision for food security allowance to entitled beneficiaries.

Penalty: If the public servant or authority fails to comply with the relief recommended by the
District Grievance Redressal Officer, penalty will be imposed by the State Food Commission
according to the provision.

https://www.insightsonindia.com/2021/06/15/national-food-security-act-nfsa/

33. Consider the following statements regarding Group 7 (G7)


1. Canada, China, Germany are the members of G7.
2. It is an intergovernmental organisation formed in 1990’s
3. The major purpose of the G-7 is to discuss and deliberate on security issues.

Which of the statements given above is/are correct?


A. 1 and 2 only
B. 2 and 3 only
C. 2 only
D. None

Correct Answer : D

Answer Justification :

All the statements given above are incorrect.

The Group of Seven (G7) is an international intergovernmental economic organization


consisting of the seven largest IMF-described advanced economies in the world: Canada,
France, Germany, Italy, Japan, the United Kingdom, and the United States. Hence,
statement 1 is incorrect.

The concept of a forum for the world's major industrialized countries emerged before
the 1973 oil crisis. On Sunday, 25 March 1973, the U.S. Secretary of the Treasury, George
Shultz, convened an informal gathering of finance ministers from West Germany Helmut
Schmidt, France Valéry Giscard d'Estaing, and the United Kingdom Anthony Barber before an
upcoming meeting in Washington, D.C. Hence, statement 2 is incorrect.

The major purpose of the G-7 is to discuss and deliberate on international economic
issues. It sometimes acts in concert to help resolve other global problems, with a special focus
on economic issues. Hence Statement 3 is incorrect.

How did G7 become G8?

1. Russia was formally inducted as a member in the group in 1998, which led G7 to become
G8.

www.insightsactivelearn.com 37
Total Marks : 200
Test-1: GS Subject
( Insta Prelims Test Series 2022 )

2. However, Russian President Vladimir Putin’s condemnable act of moving Russian troops
into eastern Ukraine and conquering Crimea in 2014 drew heavy criticism from the other G8
nations.

3. The other nations of the group decided to suspend Russia from the G8 as a consequence of
its actions and the group became G7 again in 2014.

https://www.insightsonindia.com/2021/06/14/g7-meet/

34. In which of the following areas/fields rare earth metals has its applications?
1. Ceramics
2. Catalysts
3. Glass and Polishing

Select the correct answer using the code given below


A. 1, 2 and 3
B. 1 only
C. 1 and 3 only
D. 2 only

Correct Answer : A

Answer Justification :

Rare earth metals at the heart of China-US rivalry

Context:

Recent measures of America and other countries to reduce import dependency on China for
rare earth minerals

www.insightsactivelearn.com 38
Total Marks : 200
Test-1: GS Subject
( Insta Prelims Test Series 2022 )

Background

What are rare earth minerals?

The rare earths minerals (REM) are a set of seventeen metallic elements. These
include the fifteen lanthanides on the periodic table in addition to scandium and
yttrium that show similar physical and chemical properties to the lanthanides.

The REMs have unique catalytic, metallurgical, nuclear, electrical, magnetic and
luminescent properties. While named ‘rare earth’, they are in fact not that rare and
are relatively abundant in the Earth’s crust.

Strategic importance of REM

Its usage range from daily use (e.g., lighter flints, glass polishing mediums, car
alternators) to high-end technology (lasers, magnets, batteries, fibre-optic
telecommunication cables).

Even futuristic technologies need these REMs (For example high-temperature


superconductivity, safe storage and transport of hydrogen for a post-hydrocarbon
economy, environmental global warming and energy efficiency issues).

Due to their unique magnetic, luminescent, and electrochemical properties, they


help in technologies perform with reduced weight, reduced emissions, and
energy consumption; therefore give them greater efficiency, performance,
miniaturization, speed, durability, and thermal stability

www.insightsactivelearn.com 39
Total Marks : 200
Test-1: GS Subject
( Insta Prelims Test Series 2022 )

Heavy dependence on China for REM

In 2019, US imported 80% of its rare earth minerals from China

European Union (EU) gets 98% of its supply from China

Indiahas the world’s fifth-largest reserves of rare earth elements, nearly twice as
much as Australia, but it imports most of its rare earth needs in finished form from its
geopolitical rival, China

Such high dependence on China might result in creating geopolitical issues for importing
countries if China excludes others from accessing these crucial resources.

Some of the actions taken by countries to tackle this situation

US senate passed a law recently aimed at improving American competitiveness that


includes provisions to improve critical minerals supply chain.

US also aims to boost production and processing of rare earths and lithium, another key
mineral component while “working with allies to increase sustainable global supply and
reduce reliance on competitors”

What India needs to do?

India has granted government corporations such as Indian Rare Earths Limited (IREL) a
monopoly over the primary mineral that contains REEs. However, IREL accounts for only a

www.insightsactivelearn.com 40
Total Marks : 200
Test-1: GS Subject
( Insta Prelims Test Series 2022 )

minuscule fraction of the world’s production: only 2265 tonnes of REOs in 2016-17, providing
almost no value to domestic manufacturers and consumers, who continued to import finished
REE derivatives from China.

https://www.insightsonindia.com/2021/06/14/rare-earth-metals-at-the-heart-of-china-us-rivalry/

35. Consider the following statements regarding Atlantic Charter


1. It is a joint declaration issued during World War I
2. It is the joint declaration between Germany and United States

Which of the statements given above is/are correct?


A. 1 only
B. 2 only
C. Both 1 and 2
D. Neither 1 nor 2

Correct Answer : D

Answer Justification :

Both the statements are incorrect.

Atlantic Charter:

Context:

President Joe Biden and British Prime Minister Boris Johnson recently inspected documents
related to the Atlantic Charter, a declaration signed by British Prime Minister Winston
Churchill and U.S. President Franklin D. Roosevelt in August 1941.

The two leaders plan to sign what they’re calling a new Atlantic Charter, pledging to
“defend the principles, values, and institutions of democracy and open societies.”

About Atlantic Charter:

The Atlantic Charter was a joint declaration issued during World War II (1939-45) by
the United States and Great Britain that set out a vision for the postwar world.

First announced on August 14, 1941, a group of 26 Allied nations eventually pledged
their support by January 1942.

Among its major points were a nation’s right to choose its own government, the easing of

www.insightsactivelearn.com 41
Total Marks : 200
Test-1: GS Subject
( Insta Prelims Test Series 2022 )

trade restrictions and a plea for postwar disarmament.

The document is considered one of the first key steps toward the establishment of the
United Nations in 1945.

What Was Included In The Atlantic Charter?

The Atlantic Charter included eight common principles. This includes:

1. The United States and Britain agreed not to seek territorial gains from the war, and they
opposed any territorial changes made against the wishes of the people concerned.

2. To support the restoration of self-government to those nations who had lost it during the
war.

3. People should have the right to choose their own form of government.

https://www.insightsonindia.com/2021/06/11/atlantic-charter/

36. Consider the following statements regarding Breach of privilege motion


1. The parliamentary privileges extend to the President who is also an integral part of the
Parliament.
2. The Speaker/RS chairperson is the first level of scrutiny of a privilege motion.

Which of the statements given above is/are correct?


A. 1 only
B. 2 only
C. Both 1 and 2
D. Neither 1 nor 2

Correct Answer : B

Answer Justification :

Breach of privilege motion:

Context:

CPI MP Binoy Viswam has filed a breach of privilege motion against the Lakshadweep

www.insightsactivelearn.com 42
Total Marks : 200
Test-1: GS Subject
( Insta Prelims Test Series 2022 )

Administrator Praful K. Patel for denying him permission to visit the islands.

The MP has said that the Right of a parliamentarian to move freely and meet with
the people is integral to the privilege of their position itself.

Background:

The district administration has responded saying that the MP was not allowed to visit as per
the Covid protocols in place.

What are Parliamentary Privileges?

Parliamentary Privileges are certain rights and immunities enjoyed by members of


Parliament, individually and collectively, so that they can “effectively discharge their
functions”.

1. Article 105 of the Constitution expressly mentions two privileges, that is, freedom
of speech in Parliament and right of publication of its proceedings.

2. Apart from the privileges as specified in the Constitution, the Code of Civil
Procedure, 1908, provides for freedom from arrest and detention of members under civil
process during the continuance of the meeting of the House or of a committee thereof
and forty days before its commencement and forty days after its conclusion.

Motion against breaches:

When any of these rights and immunities are disregarded, the offence is called a breach of
privilege and is punishable under law of Parliament.

A notice is moved in the form of a motion by any member of either House against
those being held guilty of breach of privilege.

Role of the Speaker/Rajya Sabha (RS) Chairperson:

The Speaker/RS chairperson is the first level of scrutiny of a privilege motion. Hence
Statement 2 is correct.

The Speaker/Chair can decide on the privilege motion himself or herself or refer it to the
privileges committee of Parliament.

If the Speaker/Chair gives consent under relevant rules, the member concerned is given
an opportunity to make a short statement.

www.insightsactivelearn.com 43
Total Marks : 200
Test-1: GS Subject
( Insta Prelims Test Series 2022 )

Applicability:

1. The Constitution also extends the parliamentary privileges to those persons who
are entitled to speak and take part in the proceedings of a House of Parliament
or any of its committees. These include the Attorney General of India.

2. The parliamentary privileges do not extend to the President who is also an


integral part of the Parliament. Article 361 of the Constitution provides for
privileges for the President. Hence Statement 1 is incorrect.

https://www.insightsonindia.com/2021/06/11/breach-of-privilege-motion/

37. Consider the following statements regarding NATO Summit


1. It is an intergovernmental military alliance.
2. It was established by Washington treaty.
3. NATO membership is open to any other European state in a position to further the principles
of this Treaty and to contribute to the security of the North Atlantic area.

Which of the statements given above is/are correct?


A. 1 and 2 only
B. 1, 2 and 3
C. 2 and 3 only
D. 2 only

Correct Answer : B

Answer Justification :

NATO Summit:

Context:

NATO Summit was held recently in Brussels. It brought together the leaders of all 30 Allied
nations.

Outcomes of the meet:

1. The heads of the NATO countries said they “remain firmly committed to NATO’s
founding Washington Treaty, including that an attack against one Ally shall be
considered an attack against us all, as enshrined in Article 5.”

www.insightsactivelearn.com 44
Total Marks : 200
Test-1: GS Subject
( Insta Prelims Test Series 2022 )

2. It included language about updating Article 5 to include major cyberattacks, which


have become a significant and growing concern.

About North Atlantic Treaty Organization:

It is an intergovernmental military alliance.

Established by Washington treaty.

Treaty that was signed on 4 April 1949.

Headquarters — Brussels, Belgium.

Headquarters of Allied Command Operations — Mons, Belgium.

Composition:

Since its founding, the admission of new member states has increased the alliance from
the original 12 countries to 30. The most recent member state to be added to NATO was
North Macedonia on 27 March 2020.

NATO membership is open to “any other European state in a position to further the
principles of this Treaty and to contribute to the security of the North Atlantic area.”

https://www.insightsonindia.com/2021/06/15/nato-summit/

38. Consider the following statements regarding Pakke tiger reserve


1. The main perennial streams in the tiger reserve are the Nameri, Khari and Upper Dikorai.
2. It falls within the Eastern Himalaya Biodiversity Hotspot.

Which of the statements given above is/are correct?


A. 1 only
B. 2 only
C. Both 1 and 2
D. Neither 1 nor 2

Correct Answer : C

www.insightsactivelearn.com 45
Total Marks : 200
Test-1: GS Subject
( Insta Prelims Test Series 2022 )

Answer Justification :

Both the statements are correct.

Pakke tiger reserve:

Pakke Tiger Reserve is also known as Pakhui Tiger Reserve.

This Tiger Reserve has won India Biodiversity Award 2016 in the category of
‘Conservation of threatened species’ for its Hornbill Nest Adoption Programme.

It is bounded by Bhareli or Kameng River in the west and north, and by Pakke
River in the east.

Neighbours: Papum Reserve Forest in Arunachal Pradesh, Assam’s Nameri National


Park, Doimara Reserve Forest and Eaglenest Wildlife Sanctuary.

The main perennial streams in the area are the Nameri, Khari and Upper Dikorai. West
of Kameng River is Sessa Orchid Sanctuary.

It falls within the Eastern Himalaya Biodiversity Hotspot.

www.insightsactivelearn.com 46
Total Marks : 200
Test-1: GS Subject
( Insta Prelims Test Series 2022 )

39. Consider the following statements regarding Food Safety and Standards Authority of India (FSSAI)
1. Ministry of Food Processing Industries is the Administrative Ministry for the implementation of
FSSAI.
2. It has been established under the Prevention of Food Adulteration Act, 1954.
3. It is responsible for protecting and promoting public health through the regulation and
supervision of food safety.

Which of the statements given above is/are correct?


A. 1 and 3 only
B. 2 and 3 only
C. 3 only
D. 1, 2 and 3

Correct Answer : C

Answer Justification :

Food Safety and Standards Authority of India (FSSAI):

Food safety regulator FSSAI has mandated food business operators to mention FSSAI licence
or registration number on cash receipts or purchase invoice with effect from October 1.

About FSSAI:

1. Food Safety and Standards Authority of India (FSSAI) is an autonomous


statutory body established under the Food Safety and Standards Act, 2006 (FSS
Act). Hence Statement 2 is incorrect.

2. Ministry of Health & Family Welfare, Government of India is the administrative


Ministry of FSSAI. Hence Statement 1 is incorrect.

3. To pursue any food related business, the owner needs to get a certificate and license
with the permission of FSSAI.

4. The Chairperson and Chief Executive Officer of Food Safety and Standards Authority of
India (FSSAI) are appointed by Government of India.

5. The Chairperson is in the rank of Secretary to Government of India.

6. FSSAI is responsible for protecting and promoting public health through the
regulation and supervision of food safety.

www.insightsactivelearn.com 47
Total Marks : 200
Test-1: GS Subject
( Insta Prelims Test Series 2022 )

40. Consider the following statements regarding Gharial


1. Gharials live in clear freshwater river systems, congregating at river bends where the water is
deeper.
2. A typical gharial will reach 12 to 15 feet in length and weigh up to 2,000 pounds.
3. It is naturally found in Pakistan and India.

Which of the statements given above is/are correct?


A. 1 and 2 only
B. 1 and 3 only
C. 2 only
D. 1, 2 and 3

Correct Answer : A

Answer Justification :

What is a gharial?

Gharials, sometimes called gavials, are a type of Asian crocodilian distinguished by their long,
thin snouts. Crocodilians are a group of reptiles that includes crocodiles, alligators, caimans,
and more.

Habitat

Gharials live in clear freshwater river systems, congregating at river bends where the water is
deeper. They’re not well-suited for land so they generally only leave the water to bask in the
sun or to nest.

Once found from Pakistan to Myanmar, the reptile's range has shrunk to two countries: India,
along the Chambal, Girwa, and Son Rivers; and Nepal, along the Narayani River.

Appearance and behavior

A typical gharial will reach 12 to 15 feet in length and weigh up to 2,000 pounds. Gharials
regulate their body temperature by basking in the sun to warm up or resting in shade or water
to cool down.

Male gharials sport a large growth on their snout called a ghara, the Hindi word for "mud pot."
Males use their gharas to vocalize and blow bubbles during mating displays. The animals
congregate to mate and make nests during the dry season, when females lay eggs in
sandbanks along slow-moving sections of water. Eggs incubate for 70 days, and hatchlings will
stay with their mothers for several weeks or even months.

https://www.nationalgeographic.com/animals/reptiles/facts/gharial

www.insightsactivelearn.com 48
Total Marks : 200
Test-1: GS Subject
( Insta Prelims Test Series 2022 )

41. Consider the following statements regarding Buddhism


1. The Buddha (and other teachers) taught orally through discussion and debate.
2. None of the Buddha’s speeches were written down during his lifetime.

Which of the statements given above is/are correct?


A. 1 only
B. 2 only
C. Both 1 and 2
D. Neither 1 nor 2

Correct Answer : C

Answer Justification :

All the above statements are correct.

The Buddha (and other teachers) taught orally – through discussion and debate. Men
and women (perhaps children as well) attended these discourses and discussed what
they heard.

None of the Buddha’s speeches were written down during his lifetime.

After his death (c. fifth-fourth century BCE) his teachings were compiled by his disciples
at a council of “elders” or senior monks at Vesali (Pali for Vaishali in present-day Bihar).

These compilations were known as Tipitaka – literally, three baskets to hold different
types of texts. They were first transmitted orally and then written and classified
according to length as well as subject matter.

The Vinaya Pitaka included rules and regulations for those who joined the sangha
or monastic order;

the Buddha’s teachings were included in the Sutta Pitaka;

and the Abhidhamma Pitaka dealt with philosophical matters.

42. Consider the following statements regarding Al-Biruni


1. Al-Biruni’s Kitab-ul-Hind was written in Persian.
2. He translated the works of Euclid (a Greek mathematician) into Sanskrit.

www.insightsactivelearn.com 49
Total Marks : 200
Test-1: GS Subject
( Insta Prelims Test Series 2022 )

3. He translated Patanjali’s work on grammar, into Arabic.

Which of the statements given above are correct?


A. 1 and 2 only
B. 2 and 3 only
C. 1 and 3 only
D. 1, 2 and 3

Correct Answer : B

Answer Justification :

Al-Biruni’s expertise in several languages allowed him to compare languages and


translate texts.

He translated several Sanskrit works, including Patanjali’s work on grammar, into


Arabic.

For his Brahmana friends, he translated the works of Euclid (a Greek


mathematician) into Sanskrit.

Al-Biruni’s Kitab-ul-Hind, written in Arabic, is simple and lucid. It is a voluminous


text, divided into 80 chapters on subjects such as religion and philosophy, festivals,
astronomy, alchemy, manners and customs, social life, weights and measures,
iconography, laws and metrology.

Hence, statement 1 is incorrect.

43. Consider the following statements regarding Ibn Battuta


1. Muhammad bin Tughlaq appointed him as the qazi or judge of Delhi.
2. According to Ibn Battuta, travelling was safe under Delhi Sultanate.

Which of the statements given above is/are correct?


A. 1 only
B. 2 only
C. Both 1 and 2
D. Neither 1 nor 2

Correct Answer : A

Answer Justification :

www.insightsactivelearn.com 50
Total Marks : 200
Test-1: GS Subject
( Insta Prelims Test Series 2022 )

Ibn Battuta’s book of travels, called Rihla, written in Arabic, provides extremely rich and
interesting details about the social and cultural life in the subcontinent in the fourteenth
century.

Travelling overland through Central Asia, Ibn Battuta reached Sind in 1333. He had
heard about Muhammad bin Tughlaq, the Sultan of Delhi, and lured by his reputation as
a generous patron of arts and letters, set off for Delhi, passing through Multan and Uch.

The Sultan was impressed by his scholarship, and appointed him the qazi or judge of
Delhi. He remained in that position for several years, until he fell out of favour and was
thrown into prison.

Ibn Battuta meticulously recorded his observations about new cultures, peoples, beliefs,
values, etc. We need to bear in mind that this globe-trotter was travelling in the
fourteenth century, when it was much more arduous and hazardous to travel
than it is today.

Travelling was also more insecure: Ibn Battuta was attacked by bands of robbers
several times.

Hence, statement 2 is incorrect.

44. Consider the following statements regarding Imperial Chola


1. The founder of the Imperial Chola line was Vijayalaya.
2. The Uttiramerur inscriptions gives a detailed account of the village administration.
3. Rajendra I completed the construction of the famous Rajarajeswara temple or Brihadeeswara
temple at Tanjore.

Which of the statements given above are correct?


A. 1 and 2 only
B. 2 and 3 only
C. 1 and 3 only
D. 1, 2 and 3

Correct Answer : A

Answer Justification :

The founder of the Imperial Chola line was Vijayalaya. He captured Tanjore from
Muttaraiyars in 815 A.D. and built a temple for Durga.

www.insightsactivelearn.com 51
Total Marks : 200
Test-1: GS Subject
( Insta Prelims Test Series 2022 )

The two famous Uttiramerur inscriptions that give a detailed account of the village
administration under the Cholas belong to his reign. After a gap of thirty years, the Cholas
regained their supremacy under Rajaraja I.

Rajaraja I (985 – 1014 A.D.)

He completed the construction of the famous Rajarajeswara temple or


Brihadeeswara temple at Tanjore in 1010 A.D. He also helped in the construction of
a Buddhist monastery at Nagapattinam.

Rajendra I (1014-1044 A.D.)

Rajendra founded the city of Gangaikondacholapuram and constructed the


famous Rajesvaram temple in that city. He also excavated a large irrigation tank
called Cholagangam on the western side of the city.

Hence, statement 3 is incorrect.

45. Consider the following statements regarding Al-Biruni’s view on caste system
1. Al-Biruni tried to explain the caste system by looking for parallels in other societies.
2. He attempted to suggest that social divisions were unique to India.
3. The conception of social pollution, intrinsic to the caste system, was according to him, in
conformity to the laws of nature.

Which of the statements given above is/are correct?


A. 1 only
B. 2 and 3 only
C. 3 only
D. 1, 2 and 3

Correct Answer : A

Answer Justification :

Al-Biruni’s description of the caste system

Al-Biruni tried to explain the caste system by looking for parallels in other
societies. He noted that in ancient Persia, four social categories were recognised: those
of knights and princes; monks, fire-priests and lawyers; physicians, astronomers and
other scientists; and finally, peasants and artisans.

www.insightsactivelearn.com 52
Total Marks : 200
Test-1: GS Subject
( Insta Prelims Test Series 2022 )

In other words, he attempted to suggest that social divisions were not unique to
India. Hence, statement 2 is incorrect.

At the same time he pointed out that within Islam all men were considered equal,
differing only in their observance of piety.

In spite of his acceptance of the Brahmanical description of the caste system, Al-Biruni
disapproved of the notion of pollution. He remarked that everything which falls into a
state of impurity strives and succeeds in regaining its original condition of purity. The
sun cleanses the air, and the salt in the sea prevents the water from becoming polluted.
If it were not so, insisted Al-Biruni, life on earth would have been impossible. The
conception of social pollution, intrinsic to the caste system, was according to
him, contrary to the laws of nature. Hence, statement 3 is incorrect.

46. In medieval India, Uluq and Dawa were

A. Grading of horses
B. Postal system
C. Banking system
D. System of hierarchy in military

Correct Answer : B

Answer Justification :

In India the postal system is of two kinds. The horsepost, called uluq, is run by royal
horses stationed at a distance of every four miles. The foot-post has three stations per
mile; it is called dawa, that is one-third of a mile ... Now, at every third of a mile there is a
wellpopulated village, outside which are three pavilions in which sit men with girded loins
ready to start. Each of them carries a rod, two cubits in length, with copper bells at the top.
When the courier starts from the city he holds the letter in one hand and the rod with its bells
on the other; and he runs as fast as he can. When the men in the pavilion hear the ringing of
the bell they get ready. As soon as the courier reaches them, one of them takes the letter from
his hand and runs at top speed shaking the rod all the while until he reaches the next dawa.
And the same process continues till the letter reaches its destination. This foot-post is quicker
than the horse-post; and often it is used to transport the fruits of Khurasan which are much
desired in India.

Hence, option (b) is correct.

www.insightsactivelearn.com 53
Total Marks : 200
Test-1: GS Subject
( Insta Prelims Test Series 2022 )

47. Arrange the following travellers from earliest to latest to arrive in India.
1. Marco Polo
2. Afanasii Nikitich Nikitin
3. Seydi Ali Reis

Select the correct answer using the code given below:


A. 1 2 3
B. 1 3 2
C. 2 1 3
D. 2 3 1

Correct Answer : A

Answer Justification :

Tenth-eleventh centuries

973-1048 Muhammad ibn Ahmad Abu Raihan al-Biruni (from Uzbekistan)

Thirteenth century

1254-1323 Marco Polo (from Italy)

Fourteenth century

1304-77 Ibn Battuta (from Morocco)

Fifteenth century

1413-82 Abd al-Razzaq Kamal al-Din ibn Ishaq al-Samarqandi (from Samarqand)

1466-72 Afanasii Nikitich Nikitin (years spent in India) (fifteenth century, from
Russia)

www.insightsactivelearn.com 54
Total Marks : 200
Test-1: GS Subject
( Insta Prelims Test Series 2022 )

Sixteenth century

1518 Duarte Barbosa, d.1521 (from Portugal) (visit to India)

1562 Seydi Ali Reis (from Turkey) (year of death)

1536-1600 Antonio Monserrate (from Spain)

Seventeenth century

1626-31 Mahmud Wali Balkhi (from Balkh) (years spent in India)

1600-67 Peter Mundy (from England)

1605-89 Jean-Baptiste Tavernier (from France)

1620-88 François Bernier (from France)

Hence, option (a) is correct.

48. Which of the following are sites of Old Stone Age?


1. Soan valley
2. Potwar Plateau
3. Siwalik hills
4. Attirampakkam near Chennai

Select the correct answer using the code given below:


A. 1, 2 and 3 only
B. 2, 3 and 4 only
C. 1, 2 and 4 only
D. 1, 2, 3 and 4

Correct Answer : D

www.insightsactivelearn.com 55
Total Marks : 200
Test-1: GS Subject
( Insta Prelims Test Series 2022 )

Answer Justification :

Paleolithic or Old Stone Age

The Old Stone Age sites are widely found in various parts of the Indian subcontinent. These
sites are generally located near water sources. Several rock shelters and caves used by the
Paleolithic people are scattered across the subcontinent. They also lived rarely in huts made of
leaves. Some of the famous sites of Old Stone Age in India are:

a. The Soan valley and Potwar Plateau on the northwest India.

b. The Siwalik hills on the north India.

c. Bhimpetka in Madhya Pradesh.

d. Adamgarh hill in Narmada valley.

e. Kurnool in Andhra Pradesh and

f. Attirampakkam near Chennai.

All the above statements are correct.

49. Consider the following statements regarding Varahamihira


1. Varahamihira declared that the earth was spherical in shape and that it rotates on its own
axis.
2. Varahamihira composed Pancha Siddhantika, the five astronomical systems.
3. His Brihadjataka is considered to be a standard work on astrology.

Which of the statements given above is/are correct?


A. 1 only
B. 2 and 3 only
C. 3 only
D. 1, 2 and 3

Correct Answer : B

Answer Justification :

Science

The Gupta period witnessed a brilliant activity in the sphere of mathematics, astronomy,
astrology and medicine. Aryabhatta was a great mathematician and astronomer. He wrote the
book Aryabhatiya in 499 A.D. It deals with mathematics and astronomy. It explains
scientifically the occurrence of solar and lunar eclipses. Aryabhatta was the first to declare
that the earth was spherical in shape and that it rotates on its own axis. However,

www.insightsactivelearn.com 56
Total Marks : 200
Test-1: GS Subject
( Insta Prelims Test Series 2022 )

these views were rejected by later astronomers like Varahamihira and Brahmagupta.
Hence, statement 1 is incorrect.

Varahamihira composed Pancha Siddhantika, the five astronomical systems.

He was also a great authority on astrology.

His work Brihadsamhita is a great work in Sanskrit literature.

It deals with a variety of subjects like astronomy, astrology, geography,


architecture, weather, animals, marriage and omens.

His Brihadjataka is considered to be a standard work on astrology.

50. Which of the following are characteristic features of the Neolithic culture?
1. A different type of stone tools called microliths were used.
2. Mud brick houses were built.
3. Polishing of stone tools
4. Large urns were used as coffins for the burial of the dead.

Select the correct answer using the code given below:


A. 1, 2 and 3 only
B. 2, 3 and 4 only
C. 1, 2 and 4 only
D. 1, 2, 3 and 4

Correct Answer : B

Answer Justification :

In the sites of Mesolithic Age, a different type of stone tools is found. These are tiny
stone artifacts, often not more than five centimeters in size, and therefore called
microliths. Hence, statement 1 is incorrect.

Neolithic Age

The chief characteristic features of the Neolithic culture are the practice of agriculture,
domestication of animals, polishing of stone tools and the manufacture of pottery.

In fact, the cultivation of plants and domestication of animals led to the emergence of

www.insightsactivelearn.com 57
Total Marks : 200
Test-1: GS Subject
( Insta Prelims Test Series 2022 )

village communities based on sedentary life.

There was a great improvement in technology of making tools and other


equipments used by man. Stone tools were now polished.

The polished axes were found to be more effective tools for hunting and cutting
trees.

Mud brick houses were built instead of grass huts.

Wheels were used to make pottery.

Pottery was used for cooking as well as storage of food grains.

Large urns were used as coffins for the burial of the dead.

There was also improvement in agriculture. Wheat, barely, rice, millet were cultivated in
different areas at different points of time. Rice cultivation was extensive in eastern India.

Domestication of sheep, goats and cattle was widely prevalent.

Cattle were used for cultivation and for transport.

The people of Neolithic Age used clothes made of cotton and wool.

51. Which of the following countries is/are surrounded by the Gulf of Aden?
1. Yemen
2. Ethiopia
3. Somalia

Select the correct answer using the code given below


A. 2 only
B. 1 and 3 only
C. 3 only
D. None

Correct Answer : B

www.insightsactivelearn.com 58
Total Marks : 200
Test-1: GS Subject
( Insta Prelims Test Series 2022 )

Answer Justification :

Gulf Of Aden:

Maiden Indian Navy – European Union Naval Force (EUNAVFOR) Exercise was held in
the Gulf Of Aden.

The Gulf of Aden, also known as the Gulf of Berbera, is a deepwater gulf between
Yemen to the north, the Arabian Sea to the east, Djibouti to the west, and the Guardafui
Channel, Socotra (Yemen), and Somalia to the south.

In the northwest, it connects with the Red Sea through the Bab-el-Mandeb strait, and it
connects with the Arabian Sea to the east. To the west, it narrows into the Gulf of
Tadjoura in Djibouti.

52. Consider the following statements regarding Hallmarking of Gold


1. India is the biggest consumer of gold.
2. The Bureau of Indian Standard (BIS) operates gold and silver hallmarking scheme in India.

Which of the statements given above is/are correct?


A. 1 only
B. 1 and 3 only
C. 2 only
D. 2 and 3 only

www.insightsactivelearn.com 59
Total Marks : 200
Test-1: GS Subject
( Insta Prelims Test Series 2022 )

Correct Answer : C

Answer Justification :

Both the statements are correct.

What is hallmarking of gold, and for whom is it now mandatory?

Context:

The government has announced the phased implementation of mandatory hallmarking of


gold jewellery with effect from June 16.

Applicability:

In the first phase, gold hallmarking will be available in only 256 districts and jewellers having
annual turnover above Rs 40 lakh will come under its purview.

What is hallmarking of gold?

Hallmarking is the “accurate determination and official recording of the proportionate


content of precious metal in precious metal articles.”

So, it is like a “guarantee of purity or fineness” of precious metal articles.

The Bureau of Indian Standard (BIS) operates gold and silver hallmarking scheme in
India.

Metals covered under the purview of hallmarking:

1. Gold jewellery and gold artefacts.

2. Silver jewellery and silver artefacts.

Exceptions:

1. Export and re-import of jewellery as per Trade Policy of Government of India — Jewellery
for international exhibitions, jewellery for government-approved B2B domestic
exhibitions.

2. Watches, fountain pens and special types of jewellery such as Kundan, Polki and Jadau.

www.insightsactivelearn.com 60
Total Marks : 200
Test-1: GS Subject
( Insta Prelims Test Series 2022 )

What was the need of making hallmarking mandatory?

1. India is the biggest consumer of gold. However, the level of hallmarked jewellery is
very low in the country- only 30% of Indian gold jewellery is hallmarked. Main reason
behind this is non-availability of sufficient assaying and hallmarking centres
(A&HC).

2. The mandatory hallmarking will protect the public against lower caratage and
ensure consumers do not get cheated while buying gold ornaments.

3. It will also help to get the purity as marked on the ornaments.

4. It will bring in transparency and assure the consumers of quality.

53. Consider the following statements regarding Neutrino


1. They are one of the most abound particles in the universe.
2. They can even pass through the earth and come out on the other side.
3. Neutrinos carry an electrical charge and are nearly massless.

Which of the statements given above is/are correct?


A. 2 only
B. 1 only
C. 1, 2 and 3
D. 1 and 2 only

Correct Answer : D

Answer Justification :

Neutrinos are abundantly found in nature. The Sun, the stars and the atmosphere produce
millions of neutrinos every second. Most of these neutrinos pass through our body and we do
not realize it. They can even pass through the earth and come out on the other side.

Neutrinos are similar to the more familiar electron, with one crucial difference:
neutrinos do not carry an electric charge. Because neutrinos are electrically neutral, they
are not affected by the electromagnetic forces which act on electrons. Neutrino belongs to the
Lepton category.

Neutrinos are affected only by a "weak" sub-atomic force of a much shorter range than
electromagnetism, and are therefore able to pass through great distances in matter without
being affected by it.

www.insightsactivelearn.com 61
Total Marks : 200
Test-1: GS Subject
( Insta Prelims Test Series 2022 )

54. Consider the following statements regarding Public Accounts Committee (PAC)
1. This committee was set up first in 1921 under the provisions of the Government of India Act of
1919
2. By convention, the chairman of the committee is selected invariably from the Opposition.
3. The PAC is formed every five years.

Which of the statements given above is/are correct?


A. 1 and 2 only
B. 2 and 3 only
C. 3 only
D. None

Correct Answer : A

Answer Justification :

Public Accounts Committee

This committee was set up first in 1921 under the provisions of the Government of
India Act of 1919 and has since been in existence. At present, it consists of 22
members (15 from the Lok Sabha and 7 from the Rajya Sabha). The members are elected by
the Parliament every year from amongst its members according to the principle of
proportional representation by means of the single transferable vote. Thus, all parties get due
representation in it.

The term of office of the members is one year. A minister cannot be elected as a member of the
committee. The chairman of the committee is appointed from amongst its members by the
Speaker. Until 1966 - ‘67, the chairman of the committee belonged to the ruling party.
However, since 1967 a convention has developed whereby the chairman of the
committee is selected invariably from the Opposition.

The committee is assisted by the CAG. In fact, the CAG acts as a guide, friend and
philosopher of the committee.

The PAC is formed every year with a strength of not more than 22 members of which
15 are from Lok Sabha and 7 from Rajya Sabha. Hence Statement 3 is incorrect.

The term of office of the members is one year.

55. Which of the following is/are the key components of Deep Ocean Mission
1. Development of Ocean Climate Change Advisory Services
2. To explore and identify potential sources of hydrothermal minerals that are sources of
precious metals formed from the earth’s crust along the Indian Ocean mid-oceanic ridges.
3. Development of a component for searching deep-sea flora and fauna, including microbes, and

www.insightsactivelearn.com 62
Total Marks : 200
Test-1: GS Subject
( Insta Prelims Test Series 2022 )

studying ways to sustainably utilise them.

Which of the statements given above is/are correct?


A. 1 only
B. 1 and 3 only
C. 1, 2 and 3
D. 2 and 3 only

Correct Answer : C

Answer Justification :

India set to launch deep-sea mission:

Context:

The Union Cabinet has approved the long pending Deep-Ocean Mission.

About the Mission:

The mission proposes to explore the deep ocean similar to the space exploration started by
ISRO about 35 years ago.

The focus of the mission will be on deep-sea mining, ocean climate change advisory
services, underwater-vehicles and underwater-robotics related technologies.

The mission is expected to cost ₹4,077 crore over the next five years.

The Ministry of Earth Sciences (MoES) will be the nodal Ministry implementing this
multi-institutional mission.

Key Components of the mission:

1. A manned submersible will be developed to carry three people to a depth of 6,000


metres in the ocean with a suite of scientific sensors and tools. An Integrated Mining
System will be developed for mining polymetallic nodules at those depths in the
central Indian Ocean.

2. Development of Ocean Climate Change Advisory Services.

3. Development of a component for searching deep-sea flora and fauna, including microbes,
and studying ways to sustainably utilise them.

www.insightsactivelearn.com 63
Total Marks : 200
Test-1: GS Subject
( Insta Prelims Test Series 2022 )

4. The next component is to explore and identify potential sources of hydrothermal


minerals that are sources of precious metals formed from the earth’s crust along the
Indian Ocean mid-oceanic ridges.

5. It has a component for studying and preparing detailed engineering designs for
offshore Ocean Thermal Energy Conversion (OTEC) powered desalination plants.

6. The final component is aimed at grooming experts in the field of ocean biology and
engineering. This component aims to translate research into industrial applications and
product development through on-site business incubator facilities.

Significance:

The mission will give a boost to efforts to explore India’s vast Exclusive Economic Zone
and Continental Shelf.

The plan will enable India to develop capabilities to exploit resources in the Central
Indian Ocean Basin (CIOB).

Potential:

India has been allotted 75,000 square kilometres in the Central Indian Ocean Basin
(CIOB) by UN International Sea Bed Authority for exploration of poly-metallic nodules.

CIOB reserves contain deposits of metals like iron, manganese, nickel and cobalt.

It is envisaged that 10% of recovery of that large reserve can meet the energy
requirement of India for the next 100 years.

What are PMN?

Polymetallic nodules (also known as manganese nodules) are potato-shaped, largely


porous nodules found in abundance carpeting the sea-floor of world oceans in deep-sea.

Composition: Besides manganese and iron, they contain nickel, copper, cobalt, lead,
molybdenum, cadmium, vanadium, titanium, of which nickel, cobalt and copper are considered
to be of economic and strategic importance.

56. Consider the following statements regarding the International Labour Organisation (ILO)
1. It was established as an agency for the League of Nations following World War I.
www.insightsactivelearn.com 64
Total Marks : 200
Test-1: GS Subject
( Insta Prelims Test Series 2022 )

2. It became the first specialised agency of the United Nations (UN) in the year 1946.
3. Global Wage Report is released by ILO

Which of the statements given above is/are correct?


A. 1, 2 and 3
B. 1 only
C. 2 only
D. 1 and 3 only

Correct Answer : A

Answer Justification :

All the statements given above are correct.

India completes term as Chair of ILO”s Governing Body:

Context:

India’s term as chair of the Governing Body of the International Labour Organisation
(ILO) ended recently (October 2020- June 2021).

Last year, India assumed the chairmanship of the Governing Body of the International
Labour Organisation after a gap of 35 years.

About the Governing Body:

It is the apex executive body of the ILO which decides policies, programmes, agenda, budget
and elects the Director-General. It meets in Geneva.

About ILO:

Established as an agency for the League of Nations following World War I.

Established by the Treaty of Versailles in 1919.

It became the first specialised agency of the United Nations (UN) in the year
1946.

It got the Nobel Peace Prize in 1969.

It is the only tripartite U.N. agency. It brings together governments, employers and

www.insightsactivelearn.com 65
Total Marks : 200
Test-1: GS Subject
( Insta Prelims Test Series 2022 )

workers.

Headquarters: Geneva, Switzerland.

Key reports:

1. World Employment and Social Outlook.

2. Global Wage Report.

https://www.insightsonindia.com/2021/06/26/india-completes-term-as-chair-of-ilos-governing-b
ody/

57. Bosporus strait connects which of the following Sea or Bays


1. Black Sea
2. Sea of Marmara
3. Azov Sea
4. Aegean Sea

Select the correct answer using the code given below:


A. 1 and 2 only
B. 1 and 4 only
C. 2 and 3 only
D. 2 and 4 only

www.insightsactivelearn.com 66
Total Marks : 200
Test-1: GS Subject
( Insta Prelims Test Series 2022 )

Correct Answer : A

Answer Justification :

Black Sea:

Black Sea is an inland sea located between far-southeastern Europe and the far-
western edges of the continent of Asia and the country of Turkey.

Bordering Countries: Romania, Bulgaria, Ukraine, Russia, Georgia, and Turkey.

It connects to the Mediterranean Sea first through the Bosporus Strait, then
through the Sea of Marmara and the Dardanelles Strait, then south through the
Aegean Sea and the Sea of Crete.

The Black Sea is also connected to the Sea of Azov by the Strait of Kerch.

www.insightsactivelearn.com 67
Total Marks : 200
Test-1: GS Subject
( Insta Prelims Test Series 2022 )

58. Pygmy hogs, sometimes seen in the news, is a/an

A. Endangered Species
B. Vulnerable Species
C. Critically Endangered Species
D. Least Concern Species

Correct Answer : C

Answer Justification :

Pygmy hogs:

Context:

Eight pygmy hogs were released in Manas National Park, Assam.

They were released by the Pygmy Hog Conservation Programme (PHCP).

By 2025, the PHCP plans to release a target of 60 pygmy hogs in Manas- their home
where their last original population still survives, albeit in declined numbers.

What is PHCP?

www.insightsactivelearn.com 68
Total Marks : 200
Test-1: GS Subject
( Insta Prelims Test Series 2022 )

In 1995, Durrell Wildlife Conservation Trust, Jersey, UK partnered with Forest Department,
Government of Assam, IUCN, Wild Pig Specialist Group and Ministry of Environment, Forest
and Climate Change, Government of India and formed PHCP which is being implemented with
the PHCP’s key partner Aaranyak and EcoSystems India.

About Pygmy hogs:

1. They are the world’s rarest and smallest wild pigs.

2. The pygmy hog is native to dense alluvial grasslands in the southern foothills of the
Himalayas.

3. Endemic to India, they are restricted to very few locations around Manas National
Park in north-western Assam.

4. With just around 250 animals in the wild, the pygmy hog is one of the world’s most
threatened mammals.

5. Currently listed as ‘Critically Endangered’ on the IUCN Red List of Threatened


Species.

6. The pygmy hog is designated as a Schedule I species in India under the Wildlife
Protection Act, 1972.

59. Gain of function, sometimes seen in the news, is related to

A. Micro organisms
B. Space Science
C. Biodiversity
D. Climate Change

www.insightsactivelearn.com 69
Total Marks : 200
Test-1: GS Subject
( Insta Prelims Test Series 2022 )

Correct Answer : A

Answer Justification :

What is gain of function?

Context:

The term ‘gain of function research’ has recently cropped up in the debate about the origins
of the Covid-19 pandemic.

What is Gain-of-function Research?

‘Gain of function’ is a field of research focused on growing generations of


microorganisms, under conditions that cause mutations in a virus.

These experiments are termed ‘gain of function’ because they involve manipulating
pathogens in a way that they gain an advantage in or through a function, such as
increased transmissibility.

Such experiments allow scientists to better predict emerging infectious diseases, and to
develop vaccines and therapeutics.

How is it carried out?

1. It involves deliberately altering an organism in the lab, altering a gene, or introducing a


mutation in a pathogen to study its transmissibility, virulence and immunogenicity.

2. This is done by genetically engineering the virus and by allowing them to grow in
different growth mediums, a technique called serial passage.

Issues related to the research:

1. Gain-of-function research involves manipulations that make certain pathogenic


microbes more deadly or more transmissible.

2. There is also ‘loss-of-function’ research, which involves inactivating mutations,


resulting in a significant loss of original function, or no function to the pathogen.

3. Gain-of-function research reportedly carries inherent biosafety and biosecurity

www.insightsactivelearn.com 70
Total Marks : 200
Test-1: GS Subject
( Insta Prelims Test Series 2022 )

risks and is thus referred to as ‘dual-use research of concern’ (DURC).

Serial passaging involves allowing the pathogen to grow under different circumstances and
then observing the changes.

Relevance to Covid-19 pandemic:

The discussion around gain of function research came back to focus recently, after a
report argued that the possibility of the virus accidentally leaking out of the Wuhan
Institute of Virology could not be entirely dismissed.

While scientists had earlier ruled out the possibility of the virus being ‘genetically
engineered’, a recent report said serial passaging may have led to the evolution of the
virus during an ongoing gain of function research project in the Chinese city.

How is it regulated in India?

All activities related to genetically engineered organisms or cells and hazardous


microorganisms and products are regulated as per the “Manufacture, Use, Import, Export
and Storage of Hazardous Microorganisms/Genetically Engineered Organisms or Cells
Rules, 1989”.

In 2020, the Department of Biotechnology issued guidelines for the establishment of


containment facilities, called ‘Biosafety labs’.

The notification provides operational guidance on the containment of biohazards and


levels of biosafety that all institutions involved in research, development and handling of
these microorganisms must comply with.

60. Consider the following statements regarding the Mansabdari system


1. It was introduced during the reign of Aurangzeb.
2. It was designated by dual representation - one by personal rank (called zat) and the other by
cavalry rank (called sawar).

Which of the statements given above is/are correct?


A. 1 only
B. 2 only
C. Both 1 and 2
D. Neither 1 nor 2

www.insightsactivelearn.com 71
Total Marks : 200
Test-1: GS Subject
( Insta Prelims Test Series 2022 )

Correct Answer : B

Answer Justification :

Instituted by the Mughal Emperor Akbar, Mansabdari was a system common to both
the military and the civil department.

Basically, the Mansabdari system was borrowed from Persia. It was prevalent during the
reign of Babur and Humayun. Akbar made some important changes to the system and
made it more efficient.

Hence, statement 1 is incorrect.

The term is derived from mansab, meaning ‘rank’. Hence, mansabdar literally means rank
holder. It was designated by dual representation - one by personal rank (called zat)
and the other by cavalry rank (called sawar).

Hence, statement 2 is correct.

61. Arrange the following mahajanapadas from west to east?


1. Koshala
2. Gandhara
3. Anga
4. Avanti

Select the correct answer using the code given below:


A. 1 2 4 3
B. 2 1 4 3
C. 2 4 1 3
D. 1 2 3 4

Correct Answer : C

Answer Justification :

www.insightsactivelearn.com 72
Total Marks : 200
Test-1: GS Subject
( Insta Prelims Test Series 2022 )

'

Hence, option (c) is correct.

62. Consider the following statements regarding Magadhas


1. Between the sixth and the fourth centuries BCE, Magadha became the most powerful
mahajanapada.
2. Initially, Pataliputra was the capital of Magadha and was shifted to Rajagaha.

Which of the statements given above is/are correct?


A. 1 only
B. 2 only
C. Both 1 and 2
D. Neither 1 nor 2

Correct Answer : A

Answer Justification :

Magadha

Between the sixth and the fourth centuries BCE, Magadha (in present-day
Bihar) became the most powerful mahajanapada. Modern historians explain this
development in a variety of ways:

Magadha was a region where agriculture was especially productive.

www.insightsactivelearn.com 73
Total Marks : 200
Test-1: GS Subject
( Insta Prelims Test Series 2022 )

Besides, iron mines (in present-day Jharkhand) were accessible and provided
resources for tools and weapons.

Elephants, an important component of the army, were found in forests in the


region.

Also, the Ganga and its tributaries provided a means of cheap and convenient
communication.

Initially, Rajagaha (the Prakrit name for present day Rajgir in Bihar) was the
capital of Magadha. Interestingly, the old name means “house of the king”. Rajagaha
was a fortified settlement, located amongst hills. Later, in the fourth century BCE,
the capital was shifted to Pataliputra, present-day Patna, commanding routes of
communication along the Ganga. Hence, statement 2 is incorrect.

63. Consider the following statements regarding Mauryan Empire


1. Chandragupta Maurya founded the Mauryan Empire.
2. Bindusara was the first ruler who inscribed his messages to his subjects and officials on stone
surfaces.

Which of the statements given above is/are correct?


A. 1 only
B. 2 only
C. Both 1 and 2
D. Neither 1 nor 2

Correct Answer : A

Answer Justification :

The growth of Magadha culminated in the emergence of the Mauryan Empire.

Chandragupta Maurya, who founded the empire (c. 321 BCE), extended control as
far northwest as Afghanistan and Baluchistan, and his grandson Asoka, arguably the
most famous ruler of early India, conquered Kalinga (present-day coastal Orissa).

Asoka was the first ruler who inscribed his messages to his subjects and officials
on stone surfaces – natural rocks as well as polished pillars. Hence, statement 2
is incorrect.

www.insightsactivelearn.com 74
Total Marks : 200
Test-1: GS Subject
( Insta Prelims Test Series 2022 )

He used the inscriptions to proclaim what he understood to be dhamma.

This included respect towards elders, generosity towards Brahmanas and those
who renounced worldly life, treating slaves and servants kindly, and respect for
religions and traditions other than one’s own.

64. Asokan pillar inscriptions have been found in which of the following places?
1. Sanchi
2. Girnar
3. Kandahar
4. Sarnath

Select the correct answer using the code given below:


A. 1 and 4 only
B. 2 and 3 only
C. 2, 3 and 4 only
D. 1, 2, 3 and 4

Correct Answer : A

Answer Justification :

Distribution of Asokan inscriptions

www.insightsactivelearn.com 75
Total Marks : 200
Test-1: GS Subject
( Insta Prelims Test Series 2022 )

Girnar and Kandahar have major rock edicts.

Hence, option (a) is correct.

65. Consider the following statements regarding Prayaga Prashasti


1. Prayaga Prashasti was composed to record the achievements of the Shaka ruler Rudradaman.
2. The Prayaga Prashasti mentions that the Sudarshana lake was built by a local governor during
the rule of the Mauryas.

Which of the statements given above is/are correct?


A. 1 only
B. 2 only
C. Both 1 and 2
D. Neither 1 nor 2

Correct Answer : D

Answer Justification :

The Prayaga Prashasti (also known as the Allahabad Pillar Inscription) was composed
in Sanskrit by Harishena, the court poet of Samudragupta, arguably the most
powerful of the Gupta rulers (c. fourth century CE), is a case in point. Hence, both
statement 1 and 2 are incorrect.

www.insightsactivelearn.com 76
Total Marks : 200
Test-1: GS Subject
( Insta Prelims Test Series 2022 )

Junagadh rock inscription of Rudradaman, also known as the Girnar Rock inscription
of Rudradaman

The Sudarshana lake was an artificial reservoir.

We know about it from a rock inscription (c. second century CE) in Sanskrit,
composed to record the achievements of the Shaka ruler Rudradaman.

The inscription mentions that the lake, with embankments and water channels,
was built by a local governor during the rule of the Mauryas.

However, a terrible storm broke the embankments and water gushed out of the
lake.

Rudradaman, who was then ruling in the area, claimed to have got the lake
repaired using his own resources, without imposing any tax on his subjects.

66. Consider the following statements regarding terms used in Ancient India
1. Masattuvan and Satthavahas was used to refer to large landowners
2. Vellalar was used to refer to successful merchants.
3. Adimai was used to refer to slaves.

Which of the statements given above is/are correct?


A. 1 only
B. 2 and 3 only
C. 3 only
D. 1, 2 and 3

Correct Answer : C

Answer Justification :

What is evident is that there was a growing differentiation amongst people engaged in
agriculture – stories, especially within the Buddhist tradition, refer to landless
agricultural labourers, small peasants, as well as large landholders. The term gahapati
was often used in Pali texts to designate the second and third categories.

The large landholders, as well as the village headman (whose position was often
hereditary), emerged as powerful figures, and often exercised control over other
cultivators.

www.insightsactivelearn.com 77
Total Marks : 200
Test-1: GS Subject
( Insta Prelims Test Series 2022 )

Early Tamil literature (the Sangam texts) also mentions different categories of
people living in the villages – large landowners or vellalar, ploughmen or
uzhavar and slaves or adimai. It is likely that these differences were based on
differential access to land, labour and some of the new technologies.

Successful merchants, designated as masattuvan in Tamil and setthis and


satthavahas in Prakrit, could become enormously rich. Hence, both statement 1
and 2 are incorrect.

67. Consider the following statements regarding Punch-marked coins


1. The first gold coins were issued in the first century CE by the Kushanas.
2. The first coins to bear the names and images of rulers were issued by the Indo-Greeks.
3. Tribal republics were denied rights to issue coins.

Which of the statements given above are correct?


A. 1 and 2 only
B. 2 and 3 only
C. 1 and 3 only
D. 1, 2 and 3

Correct Answer : A

Answer Justification :

Punch-marked coins made of silver and copper (c. sixth century BCE onwards) were
amongst the earliest to be minted and used. These have been recovered from
excavations at a number of sites throughout the subcontinent.

Attempts made to identify the symbols on punchmarked coins with specific ruling
dynasties, including the Mauryas, suggest that these were issued by kings. It is also
likely that merchants, bankers and townspeople issued some of these coins.

The first coins to bear the names and images of rulers were issued by the Indo-
Greeks, who established control over the north-western part of the subcontinent c.
second century BCE. The first gold coins were issued c. first century CE by the
Kushanas. These were virtually identical in weight with those issued by contemporary
Roman emperors and the Parthian rulers of Iran, and have been found from several sites
in north India and Central Asia.

Coins were also issued by tribal republics such as that of the Yaudheyas of
Punjab and Haryana (c. first century CE). Hence, statement 3 is incorrect.

www.insightsactivelearn.com 78
Total Marks : 200
Test-1: GS Subject
( Insta Prelims Test Series 2022 )

68. Consider the following statements regarding gotras


1. Each gotra was named after a Vedic seer and all those who belonged to the same gotra were
regarded as his descendants.
2. The members of the same gotra could not marry.
3. Under Satavahanas, women retained their father’s gotras instead of adopting names derived
from their husband’s gotra.

Which of the statements given above are correct?


A. 1 and 2 only
B. 2 and 3 only
C. 1 and 3 only
D. 1, 2 and 3

Correct Answer : D

Answer Justification :

All the above statements are correct.

One Brahmanical practice, evident from c. 1000 BCE onwards, was to classify people
(especially Brahmanas) in terms of gotras.

Each gotra was named after a Vedic seer, and all those who belonged to the
same gotra were regarded as his descendants.

Two rules about gotra were particularly important:

women were expected to give up their father’s gotra and adopt that of their
husband on marriage and

members of the same gotra could not marry.

An examination of the names of women who married Satavahana rulers


indicates that many of them had names derived from gotras such as Gotama and
Vasistha, their father’s gotras. They evidently retained these names instead of
adopting names derived from their husband’s gotra name as they were required
to do according to the Brahmanical rules.

www.insightsactivelearn.com 79
Total Marks : 200
Test-1: GS Subject
( Insta Prelims Test Series 2022 )

69. Mlechchhas, a term was which was often used in ancient India, refers to

A. Foreigners
B. Fishermen
C. Spies
D. Tribes and nomads

Correct Answer : A

Answer Justification :

Mlechchha, also spelled mleccha, people of foreign extraction in ancient India. A Sanskrit
term, mlechchha was used by the Vedic peoples, originally to indicate the uncouth and
incomprehensible speech of foreigners and then extended to their unfamiliar behaviour.
Mlechchhas were found in northwestern India, and there is reason to believe that the people
known in Akkadian as Mlakkha were the original mlechchhas.

As a mlechchha, any foreigner stood completely outside the caste system and the ritual
ambience. Thus, historically, contact with them was viewed by the caste Hindu as
polluting.

Shakas who came from Central Asia, were regarded as mlechchhas, barbarians or
outsiders by the Brahmanas.

Sometimes those who spoke non-Sanskritic languages were labelled as mlechchhas and
looked down upon.

Hence, option (a) is correct.

70. The play is set in the ancient city of Ujjayini during the reign of the King Pālaka, near the end of the Pradyota
dynasty that made up the first quarter of the fifth century BCE. The central story is that of a noble but
impoverished young Brahmin, Cārudatta, who falls in love with a wealthy courtesan or nagarvadhu,
Vasantasenā. Which of the following play is described in the above paragraph?

A. Mudrarakshasa
B. Mālavikāgnimitram
C. Malatimadhava
D. Mrichchhakatika

Correct Answer : D

www.insightsactivelearn.com 80
Total Marks : 200
Test-1: GS Subject
( Insta Prelims Test Series 2022 )

Answer Justification :

Mrichchhakatika written by Shudraka (c. fourth century CE), the hero Charudatta was
described as both a Brahmana and a sarthavaha or merchant.

Mṛcchakatika or Mrichchhakatika (The Little Clay Cart) is a ten-act Sanskrit drama


attributed to Śūdraka, an ancient playwright who is possibly from the 5th century CE, and
who is identified by the prologue as a Kshatriya king as well as a devotee of Siva who lived for
above 110 years. The play is set in the ancient city of Ujjayini during the reign of the King
Pālaka, near the end of the Pradyota dynasty that made up the first quarter of the fifth century
BCE. The central story is that of a noble but impoverished young Brahmin, Sanskrit:
Cārudatta, who falls in love with a wealthy courtesan or nagarvadhu, Sanskrit:
Vasantasenā. Despite their mutual affection, however, the couple's lives and love are
threatened when a vulgar courtier, Samsthānaka, also known as Shakara, begins to
aggressively pursue Vasantasenā.

Hence, option (d) is correct.

71. Consider the following statements regarding the land classification during Akbar’s period:
1. Polaj is a land which was annually cultivated for each crop in succession and is never allowed
to lie fallow
2. Chachar is land left out of cultivation for a time, so that it may recover its strength.
3. Parauti is land uncultivated for five years and more
4. Banjar is land that has lain fallow for three or four years.

Select the correct answer using the code given below


A. 1 only
B. 2 and 4 only
C. 3 only
D. 2, 3 and 4 only

Correct Answer : A

Answer Justification :

Emperor Akbar classified the lands and fixed different revenues to be paid by each.

• Polaj is a land which was annually cultivated for each crop in succession and is never
allowed to lie fallow.

Hence, statement 1 is correct.

• Parauti is land left out of cultivation for a time, so that it may recover its strength.

Hence, statement 3 is incorrect.

www.insightsactivelearn.com 81
Total Marks : 200
Test-1: GS Subject
( Insta Prelims Test Series 2022 )

• Chachar is land that has lain fallow for three or four years.

Hence, statement 2 is incorrect.

• Banjar is land uncultivated for five years and more.

Hence, statement 4 is incorrect.

Of the first two kinds of land, there are three classes: good, middling and bad. They add
together the produce of each sort, and the third of this represents the medium produce, one-
third part of which is exacted as royal dues.

72. Who among the following is/are nine distinguished persons of Akbar’s court who were known as Navratnas or
nine jewels?
1. Raja Man Singh
2. Tansen
3. Todar Mal
4. Ustad Mansur

Select the correct answer using the codes given below:


A. 1 and 2 only
B. 1, 2 and 3 only
C. 3 and 4 only
D. 1, 2, 3 and 4

Correct Answer : B

Answer Justification :

Nine of the Mughal emperor Akbar's courtiers were known as navaratnas (or nine
jewels) were Abul Fazl, Abdul Rahim Khan-I-Khana, Birbal, Mulla Do-Piyaza, Faizi,
Raja Man Singh, Raja Todar Mal, Faqir Azio Din and Tansen.

Ustad Mansur (flourished 1590-1624) was a seventeenth-century Mughal painter and


court artist. He grew up during the reign of Jahangir (r. 1605 - 1627) during which period
he excelled at depicting plants and animals.

73. Consider the following statements regarding the famous Aihole Inscription:

1. It was written in Prakrit script.


2. It talks about the defeat of Harshavardhana by Pulakeshin II
3. It was written by Ravikirti.

Which of the statements given above is/are correct?

www.insightsactivelearn.com 82
Total Marks : 200
Test-1: GS Subject
( Insta Prelims Test Series 2022 )

A. 1 and 2 only
B. 2 and 3 only
C. 1 and 3 only
D. 1, 2 and 3

Correct Answer : B

Answer Justification :

Aihole Inscription:

The inscription is written in Sanskrit and it is in Kannada


script.

Hence, statement 1 is incorrect.

There is a mention about the defeat of Harshavardhana by Pulakeshin II and the


victory of Chalukyas over Pallavas. It also mentions about the shifting of the capital from
Aihole to Badami.

Hence, statement 2 is correct.

They were written by Ravikirti, the court poet of Pulakeshi II who reigned from 610 to 642
CE.

Hence, statement 3 is correct.

74. Consider the following statements regarding the peasants during the Mughal period:
1. Khud-kashta term was used to refer to the non-resident cultivators who belonged to some
other village, but cultivated lands elsewhere on a contractual basis.
2. Pahi-kashta term was used to refer to the residents of the village in which they held their
lands.

Which of the statements given above is/are correct?


A. 1 only
B. 2 only
C. Both 1 and 2
D. Neither 1 nor 2

Correct Answer : D

Answer Justification :

None of the statements given above is correct.

www.insightsactivelearn.com 83
Total Marks : 200
Test-1: GS Subject
( Insta Prelims Test Series 2022 )

Sources of the seventeenth century refer to two kinds of peasants – khud-kashta and
pahi-kashta. The former were residents of the village in which they held their lands.
The latter were non-resident cultivators who belonged to some other village, but
cultivated lands elsewhere on a contractual basis. People became pahi-kashta either out
of choice – for example, when terms of revenue in a distant village were more favourable – or
out of compulsion – for example, forced by economic distress after a famine.

75. Consider the following statements regarding the Kitab-ul-Hind.


1. It was written by Abdur Razzaq Samarqandi.
2. It was written in Arabic language.

Which of the statements given above is/are correct?


A. 1 only
B. 2 only
C. Both 1 and 2
D. Neither 1 nor 2

Correct Answer : B

Answer Justification :

Al-Biruni’s Kitab-ul-Hind, written in Arabic, is simple and lucid.

Hence, statement 1 is incorrect and statement 2 is correct.

It is a voluminous text, divided into 80 chapters on subjects such as religion and philosophy,
festivals, astronomy, alchemy, manners and customs, social life, weights and measures,
iconography, laws and metrology.

76. Consider the following statements regarding Purandaradasa:


1. He was a contemporary of Kanakadasa.
2. He formulated the basic lessons of teaching Carnatic music by structuring graded exercises.
3. He introduced the raga Mayamalavagowla as the first scale to be learnt by beginners in the
field

Which of the statements given above is/are correct?


A. 1 and 2 only
B. 2 and 3 only
C. 1 and 3 only
D. 1, 2 and 3

Correct Answer : D

www.insightsactivelearn.com 84
Total Marks : 200
Test-1: GS Subject
( Insta Prelims Test Series 2022 )

Answer Justification :

All the statements given above are correct.

About Purandaradasa:

Purandara Dasa (1484–1564) was a Haridasa, great devotee of Lord Krishna and a
saint.

He was a disciple of the celebrated Madhwa philosopher-saint Vyasatirtha, and


a contemporary of yet another great Haridasa, Kanakadasa.

Purandaradasa was the pioneer who blended the rich musical streams, namely the Dravidian
and Aryan music, into a single stream known as Carnatic music.

Prior to his initiation to Haridasa tradition, Purandara Dasa was a rich merchant and was
called as Srinivasa Nayaka.

Contributions to Indian Music:

1. He formulated the basic lessons of teaching Carnatic music by structuring


graded exercises known as Svaravalis and Alankaras

2. He introduced the raga Mayamalavagowla as the first scale to be learnt by


beginners in the field – a practice that is still followed today.

3. He also composed Gitas (simple songs) for novice students.

4. He is noted for composing Dasa Sahithya, as a Bhakti movement vocalist, and a music
scholar.

Social reforms:

1. Purandara Dasa tried to reform existing practices in the society, and preached to others in
the local language, Kannada by singing devotional songs. Most of his keertanas deals with
social reform and pinpoints defects found in the society.

2. It teaches complete self-surrender and unadulterated love towards Lord Krishna, the
Supreme.

3. The philosophy of Bhakti in Purandara Dasa’s compositions stems from the essential
teachings of the realistic-pluralistic Madhwa Philosophy of Vaishnavism.

4. Purandara Dasa fought the evils of casteism through his songs. In his song
aavakulavaadarenu aavanadarenu aatma bhavavariyada mele he wonders what is the use if

www.insightsactivelearn.com 85
Total Marks : 200
Test-1: GS Subject
( Insta Prelims Test Series 2022 )

one does not understand the spirit of humanism whatever caste or status one might be
accredited to.

5. According to Purandara Dasa there were no inequalities among men and women. Both of
them had same rights and obligations in their conduct of everyday life as well as observation of
pity.

6. He made some forceful expressions on untouchability, which was dogging society.

77. Consider the following statements regarding the Bhakti saint Ravidas:
1. He was contemporary of Shankaracharya
2. He opposed caste and gender inequality
3. His devotional songs were included in the Sikh scriptures, Guru Granth Sahib.

Which of the statements given above is/are correct?


A. 2 only
B. 2 and 3 only
C. 1 and 3 only
D. 1, 2 and 3

Correct Answer : B

Answer Justification :

About Guru Ravidas:

He was a poet and saint of the Bhakti movement.

Born in Varanasi to the ‘untouchable’ leather tanners caste.

He is believed to be a disciple of the bhakti saint-poet Ramananda and a


contemporary of the bhakti saint-poet Kabir.

One of his famous disciples was the saint, Mirabai.

Ravidas’ devotional songs were included in the Sikh scriptures, Guru Granth
Sahib.

Hence, statement 3 is correct.

www.insightsactivelearn.com 86
Total Marks : 200
Test-1: GS Subject
( Insta Prelims Test Series 2022 )

The Panch Vani text of the Dadupanthi tradition within Hinduism also includes numerous
poems of Ravidas.

Ravidas taught removal of social divisions of caste and gender, and promoted
unity in the pursuit of personal spiritual freedoms.

Hence, statement 2 is correct.

Among Ravidas’s moral and intellectual achievements were the conception of


“Begampura”, a city that knows no sorrow; and a society where caste and class have
ceased to matter.

78. Sattriya dance in modern-form was introduced by:

A. Mirabai
B. Chaitanya Mahaprabhu
C. Shankaradeva
D. Sant Kabir

Correct Answer : C

Answer Justification :

Sattriya dance in modern-form was introduced by the Vaishnava saint Shankaradeva in the
15th century A.D in Assam.

Hence, option (c) is correct.

79. Which of the following was/were the works of Krishnadevaraya?


1. Manucharitam
2. Amukthamalyadha
3. Jambavati Kalyanam
4. Harikathasaram

Select the correct answer using the code given below


A. 1 and 2 only
B. 2 and 3 only
C. 3 and 4 only
D. 1 and 4 only

www.insightsactivelearn.com 87
Total Marks : 200
Test-1: GS Subject
( Insta Prelims Test Series 2022 )

Correct Answer : B

Answer Justification :

Krishna Deva Raya himself authored a Telugu work, Amukthamalyadha and Sanskrit
works, Jambavati Kalyanam and Ushaparinayam.

Allasani Peddanna was the greatest and he was called Andhrakavita Pitamaga. His
important works include Manucharitam and Harikathasaram

80. Consider the following statements regarding administration under Aurangazeb


1. He started the practice of Jarokhadarshan.
2. He reimposed jizya and pilgrim tax

Which of the statements given above is/are correct?


A. 1 only
B. 2 only
C. Both 1 and 2
D. Neither 1 nor 2

Correct Answer : B

Answer Justification :

Aurangazeb was a staunch and orthodox Muslim in his personal life. His idea was to
transform India into an Islamic state. He created a separate department to enforce moral
codes under a high-powered officer called Muhtasib. Drinking was prohibited. Cultivation and
use of bhang and other drugs were banned. Aurangazeb forbade music in the Mughal court.
He discontinued the practice of Jarokhadarshan. He also discontinued the celebration of
Dasarah and royal astronomers and astrologers were also dismissed from service.

Hence, statement 1 is incorrect.

Initially Aurangazeb banned the construction of new Hindu temples and repair of old temples.
In 1679, he reimposed jiziya and pilgrim tax. He was also not tolerant of other
Muslim sects. The celebration of Muharram was stopped.

Hence, statement 2 is correct.

81. Consider the following statements regarding Carbon dating


1. Carbon 14, a radioactive carbon isotope is present in all living objects.
2. The half-life of C14 is 5568 years.
3. Antiquities older than 70 million years can be dated by this method.

www.insightsactivelearn.com 88
Total Marks : 200
Test-1: GS Subject
( Insta Prelims Test Series 2022 )

Which of the statements given above are correct?


A. 1 and 2 only
B. 2 and 3 only
C. 1 and 3 only
D. 1, 2 and 3

Correct Answer : A

Answer Justification :

Radiocarbon dating

Radiocarbon or Carbon 14 (C14) is a radioactive carbon (isotope) that is present


in all living objects.

It decays, like all radioactive substances, at a uniform rate. When an object is living, the
process of decay of C14 is neutralized by absorption of C14 through air and food.
However, when an object ceases to be alive, its C14 content continues to decay at a
uniform rate but ceases to absorb C14 from air and food.

By measuring the loss of C14 content in an ancient object, its age can be determined.
This is because, as stated earlier, the decay of C14 takes place at a uniform rate.

It is known that the half-life of C14 is 5568 years. The half-life of a radioactive
material is defined as the period during which half the radioactive content in an object
disappears. Thus, the C14 content in an object that ceased to live 5568 years ago would
be half of what it was when it was living, and in an object which ceased to live 11,136
years ago, its C14 content would be one-fourth of that it had been when it was living.

But no antiquity older than 70,000 years can be dated by this method. Hence,
statement 3 is incorrect.

82. Consider the following statements regarding Ashokan inscriptions


1. Brahmi script was written from right to left.
2. The Brahmi script prevailed virtually all over India except for the north-western part.
3. Ashokan epigraphs were first deciphered in 1837 by James Prinsep.

Which of the statements given above are correct?

www.insightsactivelearn.com 89
Total Marks : 200
Test-1: GS Subject
( Insta Prelims Test Series 2022 )

A. 1 and 2 only
B. 2 and 3 only
C. 1 and 3 only
D. 1, 2 and 3

Correct Answer : B

Answer Justification :

Ashokan inscriptions

Most Ashokan inscriptions were engraved in the Brahmi script, which was
written from left to right, but some were also incised in the Kharoshthi script
which was written from right to left. Hence, statement 1 is incorrect.

However, the Brahmi script prevailed virtually all over India except for the
north-western part.

Greek and Aramaic scripts were employed in writing Ashokan inscriptions in Pakistan
and Afghanistan, but Brahmi continues to be the main script till the end of Gupta times.

An epigraphist can decipher most Indian inscriptions up to about the seventh century if
he has mastered Brahmi and its variations, but subsequently we notice strong regional
variations in this script.

These epigraphs were first deciphered in 1837 by James Prinsep, a civil servant in
the employ of the East India Company in Bengal.

83. Consider the following statements regarding Jatakas


1. Each birth story of Buddha is called a Jataka.
2. The Jatakas throw invaluable light on the social and economic conditions of the period
between the fifth and tenth century BC.

Which of the statements given above is/are correct?


A. 1 only
B. 2 only
C. Both 1 and 2
D. Neither 1 nor 2

www.insightsactivelearn.com 90
Total Marks : 200
Test-1: GS Subject
( Insta Prelims Test Series 2022 )

Correct Answer : A

Answer Justification :

The most important and interesting portion of the non-canonical literature is provided by
the stories of the previous births of Gautama Buddha.

It was believed that before he was actually born as Gautama, the Buddha passed through
over 550 births, in many cases in the form of animals.

Each birth story is called a Jataka, which is a folk tale.

The Jatakas throw invaluable light on the social and economic conditions of the
period between the fifth and second century BC. Hence, statement 2 is
incorrect.

They also make incidental references to political events in the age of the Buddha.

84. Consider the following statements regarding Sangam literature


1. The Sangam literature comprises about 30,000 lines of poetry arranged in eight anthologies
called Purananuru.
2. The poems are collected in groups of hundreds such as Ettuttokai.
3. There are two main groups, Patinenkil Kannakku (The Eighteen Lower Collections) and
Pattuppattu (The Ten Songs).

Which of the statements given above is/are correct?


A. 1 only
B. 2 and 3 only
C. 3 only
D. 1, 2 and 3

Correct Answer : C

Answer Justification :

Sangam literature

www.insightsactivelearn.com 91
Total Marks : 200
Test-1: GS Subject
( Insta Prelims Test Series 2022 )

This literature was produced over a period of three to four centuries by poets who
assembled in colleges patronized by chiefs and kings. Such colleges were called Sangam,
and the literature produced in these assemblies was known as Sangam literature.

The compilation of the corpus is attributed to the first four Christian centuries, although
they were really completed by the sixth century.

The Sangam literature comprises about 30,000 lines of poetry arranged in eight
anthologies called Ettuttokai. Hence, statement 1 is incorrect.

The poems are collected in groups of hundreds such as Purananuru (The Four
Hundred of the Exterior). Hence, statement 2 is incorrect.

There are two main goups Patinenkil Kannakku (The Eighteen Lower
Collections) and Pattuppattu (The Ten Songs).

The former is generally assumed to be older than the latter, and hence is
considered to be of great historical importance.

85. Which of the following species were known to Harrapans?


1. Buffalo
2. Goat
3. Deer
4. Gharial

Select the correct answer using the code given below:


A. 1 and 4 only
B. 2 and 3 only
C. 2, 3 and 4 only
D. 1, 2, 3 and 4

Correct Answer : D

Answer Justification :

Harappan culture

Archaeologists have been able to reconstruct dietary practices from finds of charred
grains and seeds.

www.insightsactivelearn.com 92
Total Marks : 200
Test-1: GS Subject
( Insta Prelims Test Series 2022 )

These are studied by archaeo-botanists, who are specialists in ancient plant remains.

Grains found at Harappan sites include wheat, barley, lentil, chickpea and sesame.
Millets are found from sites in Gujarat. Finds of rice are relatively rare.

Animal bones found at Harappan sites include those of cattle, sheep, goat,
buffalo and pig. Studies done by archaeo-zoologists or zooarchaeologists
indicate that these animals were domesticated. Bones of wild species such as
boar, deer and gharial are also found.

We do not know whether the Harappans hunted these animals themselves or obtained
meat from other hunting communities.

Bones of fish and fowl are also found.

All the above statements are correct.

86. Consider the following statements regarding Agricultural technologies under Harapan civilization
1. Terracotta models of the plough have been found at sites in Cholistan and at Banawali.
2. Evidence of a ploughed field was found at Shortughai in Afghanistan.
3. Traces of canals have been found at the Harappan site of Kalibangan.

Which of the statements given above is/are correct?


A. 1 only
B. 2 and 3 only
C. 3 only
D. 1, 2 and 3

Correct Answer : A

Answer Justification :

Agricultural technologies

While the prevalence of agriculture is indicated by finds of grain, it is more difficult to


reconstruct actual agricultural practices. Were seeds broadcast (scattered) on ploughed
lands?

Representations on seals and terracotta sculpture indicate that the bull was known, and
archaeologists extrapolate from this those oxen were used for ploughing.

www.insightsactivelearn.com 93
Total Marks : 200
Test-1: GS Subject
( Insta Prelims Test Series 2022 )

Moreover, terracotta models of the plough have been found at sites in Cholistan
and at Banawali (Haryana).

Archaeologists have also found evidence of a ploughed field at Kalibangan


(Rajasthan), associated with Early Harappan levels. Hence, statement 2 is
incorrect.

The field had two sets of furrows at right angles to each other, suggesting that two
different crops were grown together.

Most Harappan sites are located in semi-arid lands, where irrigation was probably
required for agriculture.

Traces of canals have been found at the Harappan site of Shortughai in


Afghanistan, but not in Punjab or Sind. Hence, statement 3 is incorrect.

It is possible that ancient canals silted up long ago. It is also likely that water drawn
from wells was used for irrigation.

Besides, water reservoirs found in Dholavira (Gujarat) may have been used to store
water for agriculture.

87. Consider the following statements regarding Craft Production under Harapan civilization
1. Mohenjodaro was almost exclusively devoted to craft production.
2. Metals like copper, bronze and gold were used for craft production.

Which of the statements given above is/are correct?


A. 1 only
B. 2 only
C. Both 1 and 2
D. Neither 1 nor 2

Correct Answer : B

Answer Justification :

About Craft Production

Chanhudaro is a tiny settlement (less than 7 hectares) as compared to


Mohenjodaro (125 hectares), almost exclusively devoted to craft production,
www.insightsactivelearn.com 94
Total Marks : 200
Test-1: GS Subject
( Insta Prelims Test Series 2022 )

including bead-making, shell-cutting, metal-working, seal-making and weight-


making. Hence, statement 1 is incorrect.

The variety of materials used to make beads is remarkable: stones like carnelian
(of a beautiful red colour), jasper, crystal, quartz and steatite; metals like
copper, bronze and gold; and shell, faience and terracotta or burnt clay.

Some beads were made of two or more stones, cemented together, some of stone with
gold caps.

The shapes were numerous – disc shaped, cylindrical, spherical, barrel-shaped,


segmented.

Some were decorated by incising or painting, and some had designs etched onto them.

88. Consider the following statements regarding resources during Harapan civilization
1. Lothal was near the best source of lapis lazuli.
2. Nageshwar and Balakot were areas where shell was available.
3. The inhabitants of Khetri region supplied copper to the Harappans.

Which of the statements given above is/are correct?


A. 1 only
B. 2 and 3 only
C. 3 only
D. 1, 2 and 3

Correct Answer : B

Answer Justification :

Materials from the subcontinent and beyond The Harappans procured materials for craft
production in various ways.

For instance, they established settlements such as Nageshwar and Balakot in


areas where shell was available.

Other such sites were Shortughai, in far-off Afghanistan, near the best source of
lapis lazuli, a blue stone that was apparently very highly valued, and Lothal
which was near sources of carnelian (from Bharuch in Gujarat), steatite (from
south Rajasthan and north Gujarat) and metal (from Rajasthan). Hence,

www.insightsactivelearn.com 95
Total Marks : 200
Test-1: GS Subject
( Insta Prelims Test Series 2022 )

statement 1 is incorrect.

Another strategy for procuring raw materials may have been to send expeditions to
areas such as the Khetri region of Rajasthan (for copper) and south India (for gold).
These expeditions established communication with local communities.

Occasional finds of Harappan artefacts such as steatite micro beads in these areas are
indications of such contact.

There is evidence in the Khetri area for what archaeologists call the Ganeshwar-
Jodhpura culture, with its distinctive non-Harappan pottery and an unusual
wealth of copper objects. It is possible that the inhabitants of this region
supplied copper to the Harappans.

89. Consider the following statements regarding the script of Harappans


1. Most inscriptions are short.
2. The script was written from right to left.
3. Motif was used to convey a meaning to those who could not read.

Which of the statements given above is/are correct?


A. 1 only
B. 2 and 3 only
C. 3 only
D. 1, 2 and 3

Correct Answer : D

Answer Justification :

An enigmatic script Harappan seals usually have a line of writing, probably containing
the name and title of the owner.

Scholars have also suggested that the motif (generally an animal) conveyed a
meaning to those who could not read.

Most inscriptions are short, the longest containing about 26 signs.

Although the script remains undeciphered to date, it was evidently not


alphabetical (where each sign stands for a vowel or a consonant) as it has just
too many signs – somewhere between 375 and 400.

www.insightsactivelearn.com 96
Total Marks : 200
Test-1: GS Subject
( Insta Prelims Test Series 2022 )

It is apparent that the script was written from right to left as some seals show a
wider spacing on the right and cramping on the left, as if the engraver began
working from the right and then ran out of space.

Consider the variety of objects on which writing has been found: seals, copper tools,
rims of jars, copper and terracotta tablets, jewellery, bone rods, even an ancient
signboard! Remember, there may have been writing on perishable materials too.

All the above statements are correct.

90. Consider the following statements regarding Ganas


1. Ganas or Sanghas were oligarchies.
2. Both Mahavira and the Buddha belonged to ganas.

Which of the statements given above is/are correct?


A. 1 only
B. 2 only
C. Both 1 and 2
D. Neither 1 nor 2

Correct Answer : C

Answer Justification :

All the above statements are correct.

The sixteen mahajanapadas

The sixth century BCE is often regarded as a major turning point in early Indian history.

It is an era associated with early states, cities, the growing use of iron, the development
of coinage, etc.

It also witnessed the growth of diverse systems of thought, including Buddhism and
Jainism.

Early Buddhist and Jaina texts mention, amongst other things, sixteen states known as
mahajanapadas.

Although the lists vary, some names such as Vajji, Magadha, Koshala, Kuru, Panchala,

www.insightsactivelearn.com 97
Total Marks : 200
Test-1: GS Subject
( Insta Prelims Test Series 2022 )

Gandhara and Avanti occur frequently. Clearly, these were amongst the most important
mahajanapadas.

While most mahajanapadas were ruled by kings, some, known as ganas or


sanghas, were oligarchies, where power was shared by a number of men, often
collectively called rajas.

Both Mahavira and the Buddha belonged to such ganas.

In some instances, as in the case of the Vajji sangha, the rajas probably controlled
resources such as land collectively.

91. Consider the following statements regarding the Military administration under the Vijayanagar Empire
1. The top-grade officers of the army were known as Nayaks or Poligars.
2. Quality breed horses were procured from foreign traders
3. Soldiers were usually paid in cash.

Which of the statements given above is/are correct?


A. 1 and 2 only
B. 2 and 3 only
C. 1 and 3 only
D. 1, 2 and 3

Correct Answer : D

Answer Justification :

All the statements given above are correct.

The Vijayanagar army was well-organized and efficient. It consisted of the cavalry, infantry,
artillery and elephants. High-breed horses were procured from foreign traders. The top-
grade officers of the army were known as Nayaks or Poligars. They were granted land in
lieu of their services. These lands were called amaram. Soldiers were usually paid in cash.

92. Raya Gopurams and Kalyanamandapam are the distinct feature of :

A. Chalukya Architecture
B. Pallava Architecture
C. Vijayanagara Architecure
D. Gupta Architecture

www.insightsactivelearn.com 98
Total Marks : 200
Test-1: GS Subject
( Insta Prelims Test Series 2022 )

Correct Answer : C

Answer Justification :

Vijayanagar was founded in 1336 by Harihara and Bukka of the Sangama dynasty. They were
originally served under the Kakatiya rulers of Warangal.

The chief characteristics of the Vijayanagara architecture were the construction of


tall Raya Gopurams or gateways and the Kalyanamandapam with carved pillars in the
temple premises. The sculptures on the pillars were carved with distinctive features. The
horse was the most common animal found in these pillars.

Hence, option (c) is correct.

93. Consider the following statements regarding Hampi:


1. It is located on the banks of the Kaveri River
2. It is a UNESCO world heritage site.

Which of the statements given above is/are correct?


A. 1 only
B. 2 only
C. Both 1 and 2
D. Neither 1 nor 2

Correct Answer : B

Answer Justification :

About Hampi:

1. It is a UNESCO world heritage site.

Hence, statement 2 is correct.

2. It was a part of the Mauryan Empire back in the third century BC.

3. Hampi was the capital city during the four different dynasties altogether in the
Vijayanagar city that came into existence in the year 1336 AD.

4. The Vijayanagara Empire reached unfathomable heights under the guidance of King
Krishnadeva Raya of the Tuluva Dynasty.

5. 'Kishkindha Kaand' in Ramayana has special significance concerning Hampi.

6. It is located on the banks of Tungabhadra river.

www.insightsactivelearn.com 99
Total Marks : 200
Test-1: GS Subject
( Insta Prelims Test Series 2022 )

Hence, statement 1 is incorrect.

7. By 1500 CE, Hampi-Vijayanagara was the world's second-largest medieval-era city


after Beijing, and probably India's richest at that time, attracting traders from Persia
and Portugal.

8. It has been described by UNESCO as an "austere, grandiose site" of more than 1,600
surviving remains of the last great Hindu kingdom in South India.

94. During the Medieval Indian history, the term ‘Sarraf’, was used to refer:

A. Moneylender
B. Persian Wheel
C. Village Headman
D. In-charge of ports during Akbar period

Correct Answer : A

Answer Justification :

Sarraf an Arabic word meaning money changers, brokers, bankers, coin sorters,
creditors etc in the Mughal and post-Mughal periods.

Hence, option (a) is correct.

95. Consider the following statements regarding Mahmud Gawan:


1. Under his rule, the old provinces (tarafs) were consolidated from four into two.
2. The salaries and obligations of each noble were fixed.
3. In every province, a tract of land (khalisa) was set apart for the expenses of the Sultan.

Which of the statements given above is/are correct?


A. 1 and 2 only
B. 3 only
C. 2 and 3 only
D. 1, 2 and 3

Correct Answer : C

Answer Justification :

Mahmud Gawan carried out a number of internal reforms. Some of these were aimed at
limiting the power of the nobles.

www.insightsactivelearn.com 100
Total Marks : 200
Test-1: GS Subject
( Insta Prelims Test Series 2022 )

Thus, the old provinces (tarafs) were further sub-divided from four into eight,
and the governor of each fort was to be appointed directly by the sultan.

Hence, statement 1 is incorrect.

The salaries and obligations of each noble were fixed.

For maintaining a contingent of 500 horses, a noble received a salary of 1,00,000 huns per
year. The salary could be paid in cash or by assigning a jagir. Those who were paid by means
of jagir wer allowed expenses for the collection of land revenue.

Hence, statement 2 is correct.

In every province, a tract of land (khalisa) was set apart for the expenses of the
Sultan. Efforts were made to measure the land and to fix the amount to be paid by each
cultivator to the state.

Hence, statement 3 is correct.

Mahmud Gawan was a great patron of the arts. He built a magnificent madrasa or
college in the capital, Bidar

96. Karez’ system, a water harnessing technology that originated in Iran/Persia, was built in India by:

A. Cholas
B. Bahmanis
C. Mughals
D. Hoysalas

Correct Answer : B

Answer Justification :

Introduction of Kariz in India

Karez’ system (also known as Qanat) which is a water harnessing technology that originated
in Iran/Persia.

Karez system was built by Bahmani kings in 15th Century by the Bahmani kings in Bidar,
Gulbarg and Bijapur in Karnataka and also in Burhanpur in Madhya Pradesh.

www.insightsactivelearn.com 101
Total Marks : 200
Test-1: GS Subject
( Insta Prelims Test Series 2022 )

https://www.thehindu.com/news/national/karnataka/suranga-bawadi-on-world-monument-watc
h-list/article29947854.ece

97. Consider the following pairs


Travelers Contemporary Kings
1. Domingo Paes Harshavardhan
2. Barbosa Mahmud Begarha
3. Ibn Battutah Aurangazeb

Which of the pairs given above is/are correctly matched?


A. 1 and 3 only
B. 2 only
C. 2 and 3 only
D. 1 only

Correct Answer : B

Answer Justification :

Domingo Paes was a Portuguese traveller who visited the Vijayanagara Empire,
located in the Deccan in southern India, around 1520. He was contemporary of
Vijanagara ruler Krishnadevaraya.

Hence, pair 1 is incorrectly matched.

The most famous sultan of Gujarat was Mahmud Begarha (Mahmud Shah I). Mahmud
Begarha ruled over Gujarat for more than 50 years (from 1459 to 1511).

According to a traveller, Barbosa, Mahmud, from his childhood, had been nourished
on some poison so that if a fly settled on his hand, it swelled and immediately lay dead.

Hence, pair 2 is correctly matched.

After his third pilgrimage to Mecca, Ibn Battuta decided to seek employment with the
Muslim Sultan of Delhi, Muhammad bin Tughluq.

Hence, pair 3 is incorrectly matched.

98. Consider the following historical battles:


1. Battle of Plassey
2. Third Battle of Panipat

www.insightsactivelearn.com 102
Total Marks : 200
Test-1: GS Subject
( Insta Prelims Test Series 2022 )

3. Battle of Buxar

Which of the above given events took place during the reign of Mughal emperor Shah Alam II?
A. 1 and 2 only
B. 3 only
C. 2 and 3 only
D. 1 and 3 only

Correct Answer : C

Answer Justification :

During the reign of Alamgir II(1754-1758), battle of Plassey was fought in June 1757.

Hence, statement 1 is incorrect.

During the reign of Shah Alam II(1759-1806), two decisive battles- the Third battle of
Panipat (1761) and the Battle of Buxar(1764) were fought.

Hence, statement 2 and statement 3 are correct.

99. Which of the following factors is/are the reasons for the Downfall of the Mughals?
1. The invasions of Nadir Shah and Ahmad Shah Abdali
2. The weak successors after the death of Aurangzeb
3. The financial difficulties due to continuous wars.

Select the correct answer using the codes given below:


A. 2 only
B. 1 and 3 only
C. 2 and 3 only
D. 1, 2 and 3

Correct Answer : D

Answer Justification :

All the statements given above are correct.

The period of the Great Mughals, which began in 1526 with Babur’s accession to the throne,
ended with the death of Aurangzeb in 1707. Aurangzeb’s death marked the end of an era in
Indian history. When Aurangzeb died, the empire of the Mughals was the largest in India. Yet,
within about fifty years of his death, the Mughal Empire disintegrated

The weakness of the empire was exposed when Nadir Shah imprisoned the Mughal Emperor

www.insightsactivelearn.com 103
Total Marks : 200
Test-1: GS Subject
( Insta Prelims Test Series 2022 )

and looted Delhi in 1739.

The religious and Deccan policies of Aurangazeb contributed to its decline.

The weak successors and demoralization of the Mughal army also paved the way for it the
financial difficulties due to continuous wars led to the decline.

Further, the invasions of Nadir Shah and Ahmad Shah Abdali weakened the Mughal
state.

100. Who among the following Bhakti saints propounded the Pushtimarg philosophy?

A. Shankaracharya
B. Madvacharya
C. Ramanujacharya
D. Vallabacharya

Correct Answer : D

Answer Justification :

Vallabhacharya was the founder of Suddhadvaita (Pure Non dualism) and his philosophy
is known as ‘Pushti Marg’. This sect was Krishna centered, especially his child manifestation,
and is enriched with the use of traditions, music and festivals.

www.insightsactivelearn.com 104

You might also like